Quick viewing(Text Mode)

Answer Key & Exp for GS Prelim Test-11 Held on 17 Feb' 2019

Answer Key & Exp for GS Prelim Test-11 Held on 17 Feb' 2019

th Answer Key & Exp for GS Prelim Test-11 held on 17 Feb’ 2019

1. Consider the following statements about 'Acid Igneous’ rocks: (1) They are denser than basic igneous rocks. (2) They have a high percentage of silica than that of basic igneous rocks. (3) They are darker than basic igneous rocks. Which of the statements given above is/are incorrect? (a) 1 only (b) 1 and 3 only (c) 2 and 3 only (d) 1, 2 and 3

Answer: (b) Explanation: Igneous rocks are formed by the cooling and solidification of molten rock (magma) from beneath the earth’s crust. They are normally crystalline in structure. Igneous rocks can be sub-divided on the basis of mineral composition. When they contain a high proportion of silica, they are said to be acid igneous rocks. Ex. Granite. Statement (1) is incorrect. Acid igneous rocks are less dense than basic rocks. Statement (2) is correct. Acid rocks are characterized by high silica content. Statement (3) is incorrect. Acidic igneous rocks are usually lighter in colour than basic rocks. Basic rocks have a greater proportion of basic oxides e.g. of iron, aluminium or magnesium. Hence, basic rocks are denser and darker (e.g. basalt). Value Addition: Igneous rocks begin the rock cycle, and are therefore called primary rocks. Igneous rocks are classified according to texture and mineral composition. Texture is indicative of cooling conditions. Coarse-grained texture results from slow cooling and fine-grained texture represents rapid cooling. Source: Certificate Physical and Human Geography, Goh Cheng Leong, Chapter 2: The Earth’s Crust, Page 12-13

2. Which of the following is the main reason behind aridity in desert? (a) Its continentality (b) Mountain barrier to its west (c) Its equatorial location (d) Cold current to its west

Answer: (b) Explanation: The Mountain, to the desert's west, is the primary reason for the Patagonian desert's arid status. The climatic barrier of the southern Andes is so complete, that the moisture laden westerlies hardly ever reach Patagonia. Thus, the is subjected to aridity rather than

Prelim IAS Test Series (2019) – GS Test 11 (17.02.2019) 1 Geography 1 and Current Affairs Nov 2018 continentality. The rain-shadow effect produced by great mountains can create arid areas in the leeward side of the mountains even when continentality is not particularly marked, such as in Patagonia desert. That is why, despite approximately half of the desert being only about 200 miles from the ocean, such a large desert is found in the region. The cold Falkland Current (east of desert) off the Atlantic coast of also contributes to the area's aridity. Source: Certificate Physical and Human Geography, G C Leong, Chapter 24: The Cool Temperate Eastern Margin, Page 176

3. In the context of ‘Lithosphere’, SIAL and SIMA denote ______? (a) parts of continental crust only. (b) parts of oceanic crust only. (c) parts of continental crust and oceanic crust respectively. (d) parts of oceanic crust and continental crust respectively.

Answer: (c) Explanation: A lithosphere is the rigid, outermost shell of a terrestrial-type planet. On Earth, it is composed of the crust and the portion of the upper mantle. The term 'SIAL' refers to the composition of the upper layer of the Earth's crust, namely rocks rich in Silicates and Aluminium minerals. It is sometimes equated with the continental crust because it is absent in the wide oceanic basins. As these elements are less dense than the majority of the earth's elements, they tend to be concentrated in the upper layer of the crust. The lower part is a continuous zone of denser basaltic rocks forming the ocean floors, comprising mainly Silica, Iron and Magnesium. It is therefore called SIMA. Source: Certificate Physical and Human Geography, G C Leong, Chapter 2: The Earth’s Crust, Page 12.

4. Which of the following pairs are correctly matched?

Isolines Definition

1. Isohyet : A line on the map connecting points having same amount of rainfall.

2. Isobar : A line connecting points having same atmospheric pressure.

3. Isohel : A line connecting points of same percentage of cloudiness.

4. Isotherm : A line connecting points having same amount of sunshine

Prelim IAS Test Series (2019) – GS Test 11 (17.02.2019) 2 Geography 1 and Current Affairs Nov 2018

Select the correct answer using the codes given below: (a) 1 and 2 only (b) 1 and 3 only (c) 2 and 4 only (d) 3 and 4 only

Answer: (a) Explanation: Isolines are a part of the basic terminologies regarding weather and climate. Isolines are lines drawn to link different places that share a common value. Pair 1 is correct: Isohyet is a line on a map connecting points having same amount of precipitation (rain or snow) in a given period. Pair 2 is correct: Isobar is a line on a map connecting points having same atmospheric pressure at a given time. Pair 3 is incorrect: Isohel is a line on a map connecting points having same duration of sunshine. Pair 4 is incorrect: Isotherm is a line on a map connecting points having same temperature at a given time. Value Addition: Isoneph: A line on a map connecting points that have same average percentage of cloudiness. Source: Vajiram & Ravi Yellow book - Concepts of Geography and Natural Resources - Chapter 13: Weather and Climate, Page No 102

5. Which of the following is a metamorphic rock? (a) Gneiss (b) Gabbro (c) Sandstone (d) Limestone

Answer: (a) Explanation: Metamorphic rocks are formed through the transformation of pre-existing rocks in a process known as metamorphism (meaning “change in form”). The original rock is subjected to heat and pressure which cause physical, chemical and mineralogical changes to the rock. Gneiss is a high grade metamorphic rock. It is formed by the metamorphosis of granite, or sedimentary rock. Gneiss displays distinct foliation, representing alternating layers composed of different minerals. Gabbro is an intrusive igneous rock formed from the slow cooling of magnesium-rich and iron-rich magma into a holo-crystalline mass deep beneath the Earth's surface. Much of the Earth's oceanic crust is made of gabbro, formed at mid-ocean ridges. Sandstone is a sedimentary rock composed mainly of sand-size mineral or rock grains. It is a rock made of compacted sand. Sandstone is one of the most common types of sedimentary rock and is found in sedimentary basins throughout the world.

Prelim IAS Test Series (2019) – GS Test 11 (17.02.2019) 3 Geography 1 and Current Affairs Nov 2018

Limestone is a sedimentary rock composed primarily of calcium carbonate (CaCO3) in the form of the mineral calcite. Skeletal remains of aquatic creatures contain minerals like calcite and aragonite. These are different crystal forms of the calcium carbonate (CaCO3), which make up the primary composition of limestone. Source: Certificate Physical and Human Geography, Goh Cheng Leong, Chapter 2: The Earth’s Crust, Page 13-14

6. Consider the following statements about ‘Celestial bodies’: (1) The amount of time taken by the Moon to complete a rotation on its axis is same as that it takes to revolve around the Earth. (2) When the Moon was first formed, its rotational speed and orbit were very different than they are now. Which of the statements given above is/are correct? (a) 1 only (b) 2 only (c) Both 1 and 2 (d) Neither 1 nor 2

Answer: (c) Explanation: Statement 1 is correct. If the Moon didn't spin at all, then eventually it would show its far side to the Earth while moving around our planet in its orbit. However, since the rotational period is exactly the same as the orbital period, the same portion of the Moon's sphere is always facing the Earth. Statement 2 is correct. Millions of years ago, the Moon spun at a much faster pace than it does now. However, the gravitational influence of the Earth has gradually acted upon the Moon to slow its rotation down, in the same way that the much smaller gravitational influence of the Moon acts upon the Earth to create tides. This influence slowed the period of rotation of the Moon to match that of its orbit - about 27.3 days. Source: Vajiram & Ravi Yellow book - Concepts of Geography and Natural Resources- Chapter 1: Origin and Evolution of the Earth page no: 5

7. Consider the map given below:

The mountain ranges which are shown in the map (with marks X and O) are ______

Prelim IAS Test Series (2019) – GS Test 11 (17.02.2019) 4 Geography 1 and Current Affairs Nov 2018

(a) Rockies and MacDonnell (b) Rockies and Appalachian (c) Andes and (d) Andes and

Answer: (d) Explanation: Andes: The Andes or Andean Mountains are the longest continental in the world, forming a continuous highland along the western edge of South America. The Andes mountain range is about 7,242 km long. Urals: The Urals rise like a long and narrow spine across western Russia, forming a natural divide between and . The mountain range spans 2,500 kilometres (1,550 miles) passing through tundra to the north and through forested and semi- desert landscapes to the south. Continental collisions gave rise to the Urals between 250 and 300 million years ago, making them among the oldest mountains on Earth. Source: Certificate Physical and Human Geography, Goh Cheng Leong, Chapter 2: The Earth’s Crust, Page 15

8. Fold Mountains are created when two tectonic plates of Earth are pushed towards each other. Which of the following is an example of ‘Fold Mountain’? (a) Black forest (b) Hunsruck (c) Alps (d) None of the above

Answer: (c) Explanation: ‘Fold Mountains’ are formed due to folding of the earth's crust. They are formed when two tectonic plates move together (a compressional plate margin). The movement of the two plates forces sedimentary rocks upwards into a series of folds. The Black Forest is a large forested mountain range in southwest Germany. Tectonically, the range forms a lifted fault block, which rises prominently in the west from the Upper Rhine Plain. The Hunsrück is a low mountain range in Rhineland- Palatinate, Germany. It is a block mountain. The Alps are the youngest and highest mountain system in Europe. They were created when the collided with the causing the rocks in a geosyncline to be folded up into a mountain range. Though they are not as high and extensive as other mountain systems such as the and the Andes and Rocky mountains—they are responsible for major geographic phenomena. The Alpine crests isolate one European region from another and are the

Prelim IAS Test Series (2019) – GS Test 11 (17.02.2019) 5 Geography 1 and Current Affairs Nov 2018 source of many Europe’s major rivers, such as the Rhine, Po, etc. Source: Certificate Physical and Human Geography, Goh Cheng Leong, Chapter 2: The Earth’s Crust, Page 15-17

9. Which of the following is the correct sequence of ‘Cloud Forms’ in an ascending order of their heights from the ground? (a) Cirrus-Altocumulus-Stratus (b) Stratus-Altocumulus-Cirrus (c) Stratus-Cirrus-Altocumulus (d) Altocumulus-Stratus-Cirrus

Answer: (b) Explanation: Stratus (at a height of below 2km): This is a very low cloud. It is uniformly grey and thick which appears like a low ceiling or highland fog. It brings dull weather with light drizzle. It reduces the visibility of aircraft and is thus a danger. Altocumulus (at a height in the range of 2km - 6km): These are woolly, bumpy clouds arranged in layers. They appear like waves in the blue sky. They normally indicate fine weather. Cirrus (at a height in the range of 6km - 12km): This looks fibrous and appears like wisps in the blue sky. It is often called ‘mares’ tails’. It indicates fair weather and often gives a brilliant sunset. Source: Certificate Physical and Human Geography, Goh Cheng Leong, Chapter 13: Weather, Page 101-102

10. Consider the following pairs:

Name Features

1. Doldrums : They lie between 5º N and 5º S latitude.

2. Horse latitudes : Areas of high pressure.

3. Sub-polar low pressure belt : Marked by violent storms in winter.

4. Polar high-pressure belt : Between 80ºN and 90ºN latitude.

Which of the pairs given above is/are not correctly matched? (a) 1 only (b) 2 only (c) 3 and 4 only (d) None of the above

Answer: (d) Explanation: Pairs 1, 2, 3 and 4 are correctly matched.

Prelim IAS Test Series (2019) – GS Test 11 (17.02.2019) 6 Geography 1 and Current Affairs Nov 2018

On the basis of mode of genesis pressure belts are divided into four main categories: Equatorial Low Pressure Belt: The equatorial low pressure belt is located on either side of the geographical equator in a zone extending between 5°N and 5°S latitudes. The

equatorial low pressure belt represents the zone of convergence of north-east and south- east trade winds. There are light, feeble and variable winds within this convergence belt. Because of frequent calm conditions this belt is called doldrums. Sub-Tropical High Pressure Belt: Sub-tropical high pressure belt extends between the latitudes of 30°-35° in both the hemispheres. The convergence of winds at higher altitude above this zone results in the subsidence of air from higher altitudes. Thus, descent of winds results in the contraction of their volume and ultimately causes high pressure. This is why this zone is characterized by anticyclonic conditions which cause atmospheric

Prelim IAS Test Series (2019) – GS Test 11 (17.02.2019) 7 Geography 1 and Current Affairs Nov 2018 stability and aridity. This zone of high pressure is called ‘horse latitude’ because of prevalence of frequent calms. In ancient times, the merchants carrying horses in their ships had to throw out some of the horses while passing through this zone of calm in order to lighten their ships. This is why this zone is called horse latitude. Sub-Polar Low Pressure Belts: It is located between 60°- 65° latitudes in both the hemispheres. The low pressure belt does not appear to be thermally induced. The sub- polar low pressure belt is more developed and regular in the southern hemi•sphere while it is broken in the because of over-dominance of water (oceans) in the former. This region is marked by violent storms in winter. Polar High Pressure Belt: High pressure persists at the poles throughout the year because of prevalence of very low temperature (below freezing point) all the year round. In fact, both the factors, thermal and dynamic, operate at the poles. At the North and South Poles, between 80° to 90° latitudes, the temperatures are always extremely low. The cold descending air gives rise to high pressures over the poles. Source: Vajiram & Ravi Yellow book - Concepts of Geography and Natural Resources - Chapter 14: Atmospheric Circulation, Page No 117

11. Refer the following graph showing rainfall and temperature pattern in Northern Hemisphere for the months starting from January to December: Which climate type does the graph details represent?

Prelim IAS Test Series (2019) – GS Test 11 (17.02.2019) 8 Geography 1 and Current Affairs Nov 2018

(Note: The curved line represents temperature variations and black bars show rainfall. 1 inch = 2.54 cm and 70 ° F = 21 ° C) (a) Hot and Wet Equatorial climate (b) Mediterranean climate (c) Tropical Monsoon climate (d) Laurentian climate

Answer: (b) Explanation: Mediterranean climate: The Mediterranean lands receive most of their precipitation in winter when the Westerlies shift towards equator. In the northern hemisphere, the prevailing on-shore Westerlies bring much cyclonic rain from the Atlantic to the countries bordering the . Rainfall in winter season is the most outstanding feature of the Mediterranean Climate. The rain comes in heavy showers and only on a few days with bright sunny periods between them. This is another characteristic feature of the Mediterranean winter rain. Summers are warm and bright and winters are so mild and cool. The sky is almost cloudless and sunshine is always abundant. The annual temperature range is between 15° - 25°C. Value Addition:

Prelim IAS Test Series (2019) – GS Test 11 (17.02.2019) 9 Geography 1 and Current Affairs Nov 2018

Generally, for other climatic types, maximum rain comes in summer. Source: Vajiram & Ravi Yellow book - Concepts of Geography and Natural Resources- Chapter 16: World Climatic , Page No 147

12. Consider the following statements: (1) Exfoliation is a form of mechanical weathering in which curved plates of rock are stripped away from the source rock below. (2) Exfoliation process is confined to desert regions only. Which of the statements given above is/are correct? (a) 1 only (b) 2 only (c) Both 1 and 2 (d) Neither 1 nor 2

Answer: (a) Explanation: Statement 1 is correct. Exfoliation is a form of mechanical weathering in which curved plates of rock are stripped away from rock below. It often occurs in hot areas, like deserts where the temperature is often high in the day time but eventually the temperature decreases in the night. During daytime, the rocks are exposed to the sun which causes the rocks to expand. However during the night, the rock contracts because of the cooler temperature. This process is continued and over time, the surface of the rock begins to flake off. Statement 2 is incorrect. Exfoliation process is not confined to desert areas only. They can occur in tropical regions where stresses are set up in rocks by repeated wetting and drying of the surface layers. Here, short downpours saturate the rocks and outer layers expand but when hot sun quickly dries them, they quickly shrink. Thus, outer layers of rock split off. Source: Certificate Physical and Human Geography, G C Leong, Chapter 4: Weathering, Mass Movement and Ground Water, Page 30

13. With reference to vulcanism, consider the following statements: (1) Plutonic rocks are formed over the earth’s surface. (2) Lava emerging out of earth’s surface is known as magma. (3) Basic lava is highly fluid, whereas acidic lava is highly viscous. (4) Geysers and hot springs have no relation with vulcanism Which of the statements given above is/are incorrect? (a) 1 and 2 only (b) 2, 3 and 4 only (c) 1, 2 and 4 only (d) 1, 3 and 4 only

Answer: (c) Explanation: Magma is a hot molten mass deep within the earth. Magma has properties of a liquid. When magma is allowed to cool, it crystallizes into a solid rock form. The rocks formed after the cooling of molten mass are called igneous rocks.

Prelim IAS Test Series (2019) – GS Test 11 (17.02.2019) 10 Geography 1 and Current Affairs Nov 2018

Statement 1 is incorrect. Plutonic rocks are formed when magma cools and solidifies below the earth's surface (intrusive igneous rocks), and volcanic rocks are formed when lava cools and solidifies on the earth's surface (extrusive igneous rocks). Statement 2 is incorrect. Magma that reaches the earth's surface is called lava. It may be acidic or basic. Statement 3 is correct. Basic lava is highly fluid, darker in colour and has less silica content. Acidic lava is viscous, lighter in colour and has more silica content. Statement 4 is incorrect. Hot springs are formed when superheated water flows onto the earth's surface by vulcanicity. Geysers are intermittent hot springs in which superheated water and steam are ejected to the earth's surface at regular intervals. E.g. - The famous Old Faithful geyser in Yellowstone National park, USA. Value Addition: Vulcanicity is the process through which gases and molten rock are either extruded on the earth's surface or intruded into the earth's crust. Source: Vajiram & Ravi Yellow book - Concepts of Geography and Natural Resources– Chapter 6: Volcanoes and Volcanic Landforms, Page 43.

14. With reference to ‘Ground Water’, consider the following statements: (1) Clay soils support ground water recharge (2) Pervious rock system supports surface water availability (3) The surface of saturated permeable rock is called water-table (4) The water from Artesian wells is highly suitable for agricultural or irrigation purposes Which of the statements given above is/are incorrect? (a) 1, 3 and 4 only (b) 2, 3 and 4 only (c) 1, 2 and 4 only (d) 1, 2, 3 and 4

Answer: (c) Explanation: Ground water recharge is a hydrologic process where water moves downward from the

Prelim IAS Test Series (2019) – GS Test 11 (17.02.2019) 11 Geography 1 and Current Affairs Nov 2018 earth surface towards aquifers. Recharge is the primary method through which water enters an aquifer. Porosity and permeability are among the primary factors that control the movement and storage of fluids in rocks and sediments. Porosity is a measure of how much of a rock is open space. This space can be between grains or within cracks or cavities of the rock. Permeability is a measure of the ease with which a fluid (water in this case) can move through a porous rock. Statement 1 is incorrect. Clay has very fine particles, having very less space between individual grains; hence, it can get waterlogged. It has an excellent water holding capacity, and does not allow water to percolate downwards. It actually retards ground water recharge. Statement 2 is incorrect. Pervious or Permeable rocks allow water to percolate downwards quite fast. Here, surface water availability is quite low. Statement 3 is correct. The percolated water from surface finds its way to pervious/permeable rocks (aquifer) underground. The top layer of saturated pervious rock is known as water table. Statement 4 is incorrect. The water from Artesian wells is not always suitable for agricultural or irrigation purposes as sometimes it may be hot or contain an excessive amount of mineral salts. Source: Certificate Physical and Human Geography, G C Leong, Chapter 4, Page 34-37.

15. Consider the map given below:

Prelim IAS Test Series (2019) – GS Test 11 (17.02.2019) 12 Geography 1 and Current Affairs Nov 2018

The two temperate grasslands marked (with X) in the map are ______. (a) Pampas and Velds (b) Pampas and Steppes (c) Prairies and Velds (d) Downs and Steppes

Answer: (a) Explanation: Pampas: The Pampas of South America are a grassland biome. They are flat, fertile plains. It is found primarily in Argentina and extends into Uruguay. The average temperature in the Pampas is 18° C. The climate in the pampas is humid and warm. The humid Pampas ecosystem is one of the richest grazing areas in the world. Because of its temperate climate and rich, deep soil; most of the Pampas have been cultivated and turned into croplands. Velds: The grassland regions which are located in the southern portions of the African are widely known as the Velds. Furthermore, another term given to this sub- tropical grassland is the Bushvelds. In these areas one may find fewer trees, but one will definitely find abundance shrubs, grasses and the grazing animals which do not allow for trees to have proper growth. Value Addition: Like savannas, temperate grasslands are areas of open grassland with very few trees. Temperate grasslands, however, are located in colder climate regions and receive less precipitation on average than savannas. Grassland Location

Prairies Central

Steppe Russia

Downs and

New Zealand

Prelim IAS Test Series (2019) – GS Test 11 (17.02.2019) 13 Geography 1 and Current Affairs Nov 2018

Source: Vajiram & Ravi Yellow book - Concepts of Geography and Natural Resources– Chapter 16: World Climatic Regions, Page 148

16. Which of the following tree species are commonly found in ‘Coniferous Forests’? (a) Spruce and Sal (b) Teak and Sal (c) Fir and Teak (d) Spruce and Fir

Answer: (d) Explanation: Coniferous forests are vegetation composed primarily of cone-bearing needle-leaved evergreen trees (commonly known as conifers), found in areas that have long winters and moderate to high annual precipitation. Conifers tend to be evergreen. They bear needle like leaves all year long. Some common conifers are spruces, pines and firs. Spruce pulp is important in the paper industry, and timber from the trees is used in a variety of applications. Firs are evergreen coniferous trees. Old fir trees have thick, ridged bark, and younger firs have thin, smooth bark. Teak is a tropical hardwood tree species. They are found in dry deciduous forests. Teak is known as a high-quality timber species with high resistance to decay and insect damage Sal is found in dry and wet deciduous forests of India. Also, they are found in Tropical Zone at an elevation of 500 to 1,600 meters. For example: Himalayan Vegetation. Oaks are available in a wide range of habitats from Mediterranean semi-desert to subtropical rainforest. Oak is a very important tree species of the temperate broadleaved Himalayan forests. Source: Certificate Physical and Human Geography, Goh Cheng Leong, Chapter 23: The Cool Temperate Continental Climate, Page 172

17. Which of the following is a unique climatic feature of the ‘Savanna’ region? (a) Rainfall in winter (b) An extremely short dry season (c) Alternate dry and wet seasons (d) Rainfall throughout the year

Prelim IAS Test Series (2019) – GS Test 11 (17.02.2019) 14 Geography 1 and Current Affairs Nov 2018

Answer: (c) Explanation: The Savanna climate is also called the tropical wet and dry climate. It is the climate experienced in tropical grassland regions of the world. These places are located near the equator, and they lie between the Southern and the Northern Tropics. The climate dominates many parts of the African continent, the northern region of South America and parts of Australia. The Savanna climate has alternating dry and wet seasons. Such climate is relatively hot since they lie within the tropical latitudes. Throughout the year, mean monthly temperatures soar above 18 °C. The dry season in Savanna grasslands is cooler than the wet season by a few degrees. Precipitation in the Savanna climate is mainly in the form of rainfall. Rain falls mainly in summer. The rains fall from May to September in the northern hemisphere and from October to March in the . As one moves away from the equator, the mean annual rainfall decreases. Source: Vajiram & Ravi Yellow book - Concepts of Geography and Natural Resources - Chapter 16: World Climatic Regions, Page No 144

18. Consider the following statements regarding ‘Extra-tropical cyclones’: (1) Extra-tropical cyclones have a clear frontal system which is absent in tropical cyclones. (2) Extra-tropical cyclones move from east to west whereas tropical cyclones move from west to east. Which of the statements given above is/are correct? (a) 1 only (b) 2 only (c) Both 1 and 2 (d) Neither 1 nor 2

Answer: (a) Explanation: Statement 1 is correct: Tropical cyclone has a low-pressure centre, a closed low-level atmospheric circulation, strong winds, and a spiral arrangement of thunderstorms that produce heavy rain. Extra- tropical Cyclone (temperate cyclone) occurs in the middle latitudes of the Earth. Extra- tropical cyclones

Prelim IAS Test Series (2019) – GS Test 11 (17.02.2019) 15 Geography 1 and Current Affairs Nov 2018 are low pressure systems with associated cold fronts, warm fronts, and occluded fronts. Statement 2 is incorrect: Tropical cyclones moves from east to west direction under the influence of trade winds on the other hand extra-tropical cyclones (temperate cyclones) move from west to east direction under the influence of westerly winds. Source: Vajiram & Ravi Yellow book - Concepts of Geography and Natural Resources- Chapter 15: Air Mass, Fronts and Cyclones, Page No 131

19. Knick point is associated with ______? (a) Fluvial Topography (b) Karst Topography (c) Arid Topography (d) Glaciated Topography

Answer: (a) Explanation: Over a long time, a river forms generally a smooth profile. In terms of transporting water and sediment, it is the most efficient profile to have. Typically, a river will always try to form a concave shaped profile. From time to time, certain events may occur (ex. uplift of land or fall in sea level) which can change and de-stabilise this equilibrium. Such events cause the river to vertically erode its channel so as to re-establish its long, smooth concave profile. This renewed period is known as rejuvenation. Knick point is a feature related to River Rejuvenation. A knick point is a sharp break of slope in the smooth, concave long profile of a river. At this point, vertical erosion associated with rejuvenation is at its greatest. Old and rejuvenated profiles of a river meet at the knick point. Over time the knick point will retreat upstream like a waterfall. Source: Certificate Physical and Human Geography, G C Leong, Chapter 5, Page 44.

20. Ozone is one of the most dynamic constituents of the earth’s atmosphere. Consider the following statements about ozone: (1) Stratospheric ozone supports life on the Earth. (2) Tropospheric ozone is hazardous to people’s health. Which of the statements given above is/are correct? (a) 1 only (b) 2 only (c) Both 1 and 2 (d) Neither 1 nor 2

Answer: (c) Explanation: Ozone (O3) is a reactive gas that exists in two layers of the atmosphere: the stratosphere and the troposphere. Statement 1 is correct. Ozone layer of the stratosphere acts as a filter for the highly hazardous ultraviolet radiation (UVR) from the sun. This layer protects life on the Earth from UV radiation’s harmful effects. (Note: High levels of solar UV radiations can result in very high rates of skin cancer within a population). Statement 2 is correct. Tropospheric ozone is a major atmospheric pollutant. There is a great deal of evidence to show that ground-level ozone can harm human lung functions

Prelim IAS Test Series (2019) – GS Test 11 (17.02.2019) 16 Geography 1 and Current Affairs Nov 2018 and affect the respiratory system. Exposure to ozone is linked to premature death, asthma, bronchitis, heart attack, etc. (Note: Tropospheric ozone also acts as a greenhouse gas). Source: Vajiram & Ravi Yellow book - Concepts of Geography and Natural Resources- Chapter 11: Atmosphere Composition and Structure, Page No.87

21. Consider the following statements with respect to westerly ‘Jet Streams’: (1) They are currents of fast-moving air found in the upper atmosphere. (2) They usually blow from west to east due to the rotation of the earth. (3) They are found in the northern hemisphere only. Which of the statements given above is/are correct? (a) 1 and 2 only (b) 2 and 3 only (c) 1 and 3 only (d) 1, 2 and 3

Answer: (a) Explanation: Jet streams are narrow variable band of very strong predominantly westerly air currents encircling the globe several miles above the earth. Statements 1 and 2 are correct: Jet streams are relatively narrow bands of strong wind in the upper layers of the atmosphere. Generally the wind blows from west to east in jet streams but the flow often shifts to the north and south. Statement 3 is incorrect: Jet streams are found in both - northern and southern hemispheres. For eg. subtropical jet and polar jet occurs in both the hemispheres. Value addition: The strong and rapidly moving circumpolar westerly air circulation in a narrow belt of a few hundred kilometres width in the upper limit of tropo•sphere is called jet stream. Jet stream was discovered during second ‘World War’ when American jet bomber fighter planes while flying towards Japan (from east to west) found obstructions of an air circulation which was moving in opposite direction (west to east) resulting into marked reduction in the velocity of jet fighter planes. These planes registered marked

Prelim IAS Test Series (2019) – GS Test 11 (17.02.2019) 17 Geography 1 and Current Affairs Nov 2018 increase in their velocity while they used to return (west to east) to their bases. Source: Vajiram & Ravi Yellow book - Concepts of Geography and Natural Resources- Chapter 14: Atmospheric Circulation, Page No 125

22. In the context of ‘Temperature Inversion’, consider the following statements: (1) It occurs mostly on a long winter night. (2) It is aided by the presence of clear and cloudless sky. Which of the statements given above is/are correct? (a) 1 only (b) 2 only (c) Both 1 and 2 (d) Neither 1 nor 2

Answer: (c) Explanation: Under normal conditions, air temperature usually decreases at an average rate of 6.5°C per 1000 metres (normal lapse rate) as we go up in the troposphere. Sometimes this normal trend is reversed under special circumstances i.e. air temperature increases upwards up to a few kilometres from the earth’s surface (This is called negative lapse rate). Thus, warm air layer lies over cold air layer. This phenomenon is called inversion of temperature. Such situation may occur near the earth’s surface or at greater height in the troposphere. Temperature inversion plays an important role in determining cloud forms, precipitation, and visibility. Statement 1 is correct. A long winter night provides an ideal condition for inversion of temperature. The loss of heat by terrestrial radiation from the ground surface during night exceeds the amount of insolation received from the sun through incoming shortwave electromagnetic radiation waves. Thus, the ground surface becomes too cold. The air coming in contact with the cool ground surface also becomes cold while the air layer lying above is relatively warm. Consequently, temperature inversion develops because of cold air layer below and warm air layer above. Statement 2 is correct. A ground inversion develops when air is cooled by contact with a colder surface until it becomes cooler than the overlying atmosphere. This occurs most

Prelim IAS Test Series (2019) – GS Test 11 (17.02.2019) 18 Geography 1 and Current Affairs Nov 2018 often on clear and cloudless nights. When the sky is clear and cloudless, the loss of heat through terrestrial radiation proceeds more rapidly without any obstruction. (Note: Generally, clouds absorb terrestrial radiation and hence retard loss of heat from the earth’s surface.) Source: Vajiram & Ravi Yellow book - Concepts of Geography and Natural Resources- Chapter 12: Solar Radiation, Heat Balance and Temperature, Page No. 99

23. The Summer and Winter Solstice in Northern hemisphere coincides with ______and ______, respectively in Southern Hemisphere. (a) Longest day, Shortest night (b) Shortest day, Longest night (c) Shortest day, Shortest night (d) Longest day, Longest night

Answer: (c) Explanation: During summer solstice, northern hemisphere experiences longest day and shortest night, this translates to shortest day and longest night in southern hemisphere. Similarly, during winter solstice, northern hemisphere experiences shortest day and longest night, which translates to longest day and shortest night in southern hemisphere. Source: Certificate Physical and Human Geography, G C Leong, Chapter 1, Page 5.

24. Albedo is crucial for maintaining the temperature of the Earth constant. Arrange the ecosystems mentioned below in the increasing order of their albedo: (1) Conifer Forest (2) Open Ocean (3) Hot Desert (4) Polar regions Select the correct answer using the code given below: (a) 2 – 1 – 3 – 4 (b) 1 – 2 – 4 – 3 (c) 1 – 2 – 3 – 4 (d) 2 – 1 – 4 – 3

Answer: (a) Explanation: The fraction of the incident radiation that is reflected from the earth’s surface is called ‘Albedo’. It plays an important role in the energy balance of the earth’s surface. The albedo value ranges from 0 to 1. The value of 0 refers to a blackbody, a theoretical body that absorbs 100% of the incident radiation.

Prelim IAS Test Series (2019) – GS Test 11 (17.02.2019) 19 Geography 1 and Current Affairs Nov 2018

Albedo ranging from 0.1–0.2 refers to dark-coloured, rough soil surfaces, while the values around 0.4–0.5 represent smooth, light-coloured soil surfaces. The albedo of snow cover, especially the fresh, deep snow, can reach as high as 0.9. The value of 1 refers to an ideal reflector surface in which all the energy falling on the surface is reflected. The mean albedo of the earth system is 0.30 to 0.42. Open Ocean 0.06

Conifer Forest 0.15

Hot Desert 0.4

Polar regions 0.8

Source: Vajiram & Ravi Yellow book - Concepts of Geography and Natural Resources- Chapter 12: Solar Radiation, Heat Balance and Temperature, Page No. 92

25. Anemometer is used to measure ______, which can also be alternatively determined using ______. (a) , Mercalli scale. (b) Distance, Light years. (c) Wind speed, Beaufort scale. (d) Pressure, Barometer.

Answer: (c) Explanation: An anemometer is a device used for measuring wind speed. It is a common weather station instrument. The Beaufort scale is an empirical measure that relates wind speed to observed conditions at sea or on land. Its full name is the Beaufort wind force scale. Source: Certificate Physical and Human Geography, G C Leong, Chapter 13, Page 99.

26. The Prime Meridian does not pass through which of the following countries? (a) Algeria (b) France (c) Niger (d) Spain

Answer: (c) Explanation: The Prime Meridian (0° longitude) passes through the following countries:

Prelim IAS Test Series (2019) – GS Test 11 (17.02.2019) 20 Geography 1 and Current Affairs Nov 2018

In 1884, the Greenwich meridian was adopted as the prime meridian, the imaginary north- south line where the longitude is 0 degrees. It divides the Earth into the Eastern and Western Hemispheres. The Greenwich Meridian (or prime meridian) is a 0° line of longitude from which we measure 180° to the west and 180° to the east. Source: (1) Vajiram & Ravi Yellow book - Concepts of Geography and Natural Resources- Chapter 2: About The Earth; Page no 13 (2) https://gisgeography.com/prime-greenwich-meridian/

27. Clouds are formed due to condensation of water vapour. Consider the following statements about the ‘Nimbostratus clouds’: (1) They are dark and dull clouds. (2) They bring continuous rain. Which of the statements given above is/are correct? (a) 1 only (b) 2 only (c) Both 1 and 2 (d) Neither 1 nor 2

Answer: (c)

Prelim IAS Test Series (2019) – GS Test 11 (17.02.2019) 21 Geography 1 and Current Affairs Nov 2018

Explanation: Nimbostratus clouds come under ‘Low Clouds’ category. They occur at a height of 2km (or below) in the sky. Statement 1 is correct. Nimbostratus clouds generally occur in temperate latitudes. Such clouds form a dense and extensive grey layer at low altitude. They are thick, dark, dull, amorphous and solid in appearance. Statement 2 is correct. Nimbostratus clouds are often associated with continuous rainfall. They bring dull and gloomy wet days. Precipitation (snow or rain) is prolonged and widespread. Source: Vajiram & Ravi Yellow book - Concepts of Geography and Natural Resources- Chapter 13: Weather anc Climate; Page No. 106.

28. When it is a mid-day at Greenwich, England; local time at another place ‘X’ is 5 p.m. At which one of the following longitudes, the place ‘X’ is most likely located: (a) 75° E (b) 75° W (c) 150° E (d) 150° W

Answer: (a) Explanation: According to the question, when it is mid-day at Greenwich, it is 5 o’clock - the evening time at the given location ‘X’. It means that the place is ahead of Greenwich Time. Hence, that place is located in the East of Greenwich. Difference of time is 5 hours. Now  1 hour= 60 minutes  5 hours= 5x60=300 minutes  4 minutes=1°  300 minutes= (300x1)/4=75° As this place is located in the east of Greenwich longitude, the place X is located at 75° E. Source: Vajiram & Ravi Yellow book - Concepts of Geography and Natural Resources- Chapter 2: About The Earth; Page no 13

29. Which of the following are examples of ‘Hot Local Winds’? (1) Sirrocco (2) Bora (3) Harmattan (4) Chinook Select the correct answer using the code given below: (a) 1 and 2 only (b) 1, 3 and 4 only (c) 2, 3 and 4 only (d) 1, 2, 3 and 4

Answer: (b) Explanation:

Prelim IAS Test Series (2019) – GS Test 11 (17.02.2019) 22 Geography 1 and Current Affairs Nov 2018

Winds, caused by local factors and confined to a limited area compared to planetary winds, are called local winds. They are influenced by local geography. Nearness to an ocean, lake, or mountain range can affect local winds. Point 1 is correct. Sirocco is a warm, dry and dusty (full of sands) local wind which blows in northerly direction from Sahara Desert and after crossing over the Mediterranean Sea reaches Italy, Spain etc. Point 2 is incorrect. Bora is an extremely cold and dry north-easterly wind which blows along the shore of the Adriatic Sea. Bora becomes more effective in north Italy where it descends through the southern slopes of the Alps and blow in southerly direction. Point 3 is correct. The warm and dry winds blowing from north•east and east to west in the eastern parts of Sahara desert are called Harmattan. These winds become extremely dry because of their journey over Sahara desert. While blowing over Sahara desert these winds pick up more sands mainly red sands. Point 4 is correct. Warm and dry local winds blowing on the leeward sides of the mountains are called ‘Chinook’ in the USA and foehn in Switzerland. These local vertical winds are of cyclonic origin and largely influence the weather conditions of the affected areas locally. The winds associated with the cyclones after descending through the eastern slopes of the Rockies become warm and dry and thus give birth to chinook. Source: Vajiram & Ravi Yellow book - Concepts of Geography and Natural Resources- Chapter 14: Atmospheric Circulation, Page No. 124

30. The distribution of precipitation across the globe is highly uneven. In this context, consider the following statements? (1) In tropical region, the average precipitation over land decreases from east to west. (2) In temperate region, the average precipitation over land decreases from west to east. Which of the statements given above is/are correct? (a) 1 only (b) 2 only (c) Both 1 and 2 (d) Neither 1 nor 2

Answer: (c) Explanation: The yearly precipitation averaged over the whole Earth is about 100 cm, but this is distributed very unevenly. The regions of highest rainfall are found in the monsoon areas (of ) and the equatorial zone. Statement 1 is correct. In tropical region, the average precipitation over land decreases from east to west. As the moisture-laden trade winds flows from east to west, it causes more precipitation on east coast of a continent and subsequent depreciation in rainfall thereafter. Statement 2 is correct. In temperate region, the average precipitation over land decreases from west to east. As the moisture-laden westerlies flows from west to east, it causes more precipitation on west coast of a continent and subsequent depreciation in rainfall thereafter.

Prelim IAS Test Series (2019) – GS Test 11 (17.02.2019) 23 Geography 1 and Current Affairs Nov 2018

Source: Vajiram & Ravi Yellow book - Concepts of Geography and Natural Resources- Chapter 13: Weather and Climate; Page No. 110

31. Which of the following pairs are correctly matched?

Volcano Type Example

1. Extinct Volcano : Mt. Kulal

2. Dormant Volcano : Mt. Kilimanjaro

3. Active Volcano : Mt. Etna

Select the correct answer using the code given below: (a) 1 and 2 only (b) 1 and 3 only (c) 2 and 3 only (d) 1, 2 and 3

Answer: (d) Explanation: Pair (1) is correct: Volcanoes that have not erupted at all in historic times but retain the features of volcanoes are termed as extinct volcanoes. Example: Mount Kulal is an extinct volcano located in northern Kenya. Pair (2) is correct: Dormant volcano is the one which has erupted in the past and still shows signs of possible eruption in the future. Violent eruptions of dormant volcanoes are generally preceded and accompanied by , some of which have been very destructive. Example: Mount Kilimanjaro is a dormant volcano in Tanzania. Pair (3) is correct: Active volcano is the one which erupt frequently. Most of the active volcanoes are found in the Circum-Pacific Belt which is known as ‘’. Example: Mount Etna is an active stratovolcano on the east coast of Sicily, Italy Source: Vajiram & Ravi Yellow book - Concepts of Geography and Natural Resources- Chapter 6: Volcanoes and Volcanic Landforms; Page no 45-46

Prelim IAS Test Series (2019) – GS Test 11 (17.02.2019) 24 Geography 1 and Current Affairs Nov 2018

32. Refer the following map:

The areas marked (X) in the given world map have reserves of which one of the following resources? (a) Biogenic gas reserves (b) Uranium reserves (c) Shale gas reserves (d) Iron ore reserves

Answer: (c) Explanation:  Shale gas is the name for natural gas trapped within sedimentary shale rocks. Unlike conventional gas, shale gas does not accumulate in large volumes underneath impermeable “trap” rocks. It is therefore referred to as an unconventional gas, along with other types of unconventional gas resources, like coal-bed methane and tight gas.

Prelim IAS Test Series (2019) – GS Test 11 (17.02.2019) 25 Geography 1 and Current Affairs Nov 2018

 Shale gas has become an increasingly important source of natural gas in the United States over the past decade, and interest has spread to potential gas shales in Canada, Europe, Asia, and Australia.  North America leads the worldwide production of shale gas, with the US and Canada having significant levels. Beyond the US and Canada, shale gas is so far produced at a commercial scale in Argentina and China. Source: Vajiram & Ravi Yellow book - Concepts of Geography and Natural Resources, Chapter: Natural Resources, Chapter 1: Natural Resources; Page no 191

33. The Bergeron process is related to which of the following atmospheric phenomena __? (a) Precipitation (b) Jet Streams (c) Local winds (d) Smog

Answer: (a) Explanation:  The Bergeron process generates most of the mid-latitude precipitation and relies on super cooling and super-saturation. The Bergeron effect is the continuous transfer of water vapour from super-cooled droplets to ice crystals. This is due to the difference in relative humidity around the super-cooled water and the ice.  Bergeron Process takes place when ice crystals form high in the cloud tops. These small, often microscopic ice crystals attract more water vapour, causing them to increase in size. As the ice crystals increase in size, the vapour pressure drops. This allows surrounding water droplets to evaporate, becoming smaller and smaller as the ice crystals grow.  Eventually these ice crystals become large and heavy enough that they begin to fall towards the Earth’s surface. As they do, they pass through the lower warmer portion of clouds, attracting even more water vapour and growing still larger. These ice crystals can then fall to the surface of the Earth as snow, or they can melt, becoming rain drops. Source: Vajiram & Ravi Yellow book - Concepts of Geography and Natural Resources- Chapter 13: Weather and Climate, Page No. 108

34. The International Space Station (ISS) is a habitable artificial satellite. With reference to ISS, consider the following statements: (1) ISS is located in Mesosphere layer of Earth's atmosphere. (2) ISS orbits in Low-Earth orbit. Which of the statements given above is/are correct? (a) 1 only (b) 2 only (c) Both 1 and 2 (d) Neither 1 nor 2

Answer: (b)

Prelim IAS Test Series (2019) – GS Test 11 (17.02.2019) 26 Geography 1 and Current Affairs Nov 2018

Explanation: The International Space Station (ISS) is a habitable artificial satellite jointly managed by five participating space agencies: NASA, Roscosmos, Japan Aerospace Exploration Agency, European Space Agency, and Canadian Space Agency. Statement 1 is incorrect. The International Space Station (ISS) is placed in Thermosphere. The thermosphere is the layer of the Earth's atmosphere directly above the mesosphere and below the exosphere. Statement 2 is correct. The International Space Station (ISS) orbits in Low-Earth orbit. A low Earth orbit (LEO) is an orbit around Earth with an altitude above Earth's surface of 160km - 2,000 km and an orbital period between about 84 and 127 minutes. Objects below approximately 160 km will experience very rapid orbital decay and altitude loss. Source: Vajiram & Ravi Yellow book - Concepts of Geography and Natural Resources- Chapter 11: Atmosphere, Composition and Structure; Page No. 89

35. The Tropic of Capricorn passes through which of the following countries? (a) Peru (b) Zimbabwe (c) Argentina (d) Kenya

Answer: (c) Explanation: The Tropic of Capricorn is an imaginary line of latitude going around the Earth at approximately 23.5° south of the equator.

It is the southernmost point on Earth where the sun's rays can be directly overhead at local noon. It is also one of the five major circles of latitude dividing the Earth (the others are the Tropic of Cancer in the northern hemisphere, the equator, the Arctic Circle and the Antarctic Circle). The Tropic of Capricorn runs through 10 countries: Namibia, Botswana, South , Mozambique, Madagascar, Australia, Chile, Argentina, Paraguay, and Brazil. Source: Orient Blackswan School Atlas, Page no. 81

Prelim IAS Test Series (2019) – GS Test 11 (17.02.2019) 27 Geography 1 and Current Affairs Nov 2018

36. A front which is formed when cold front over•takes warm front and warm air is completely displaced from the ground surface is called as ______: (a) Stationary front (b) Warm front (c) Cold front (d) Occluded front

Answer: (d) Explanation: Front is that sloping boundary which separates two opposing air masses having

contrasting characteristics in terms of air temperature, humidity, density, pressure, and wind direction. Stationary Front: Stationary front is formed when two contrasting air masses converge in such a way that they become parallel to each other and there is no ascent of air. In fact, the surface position of stationary front does not move either forward or backward. Warm front: Warm front is that gently sloping frontal surface along which warm and light air becomes active and aggressive and rises slowly over cold and dense air. Cold front: Cold front is that sloping frontal surface along which cold air becomes active and aggressive and invades the warm air territory and being denser remains at the ground but forcibly uplifts the warm and light air. Occluded Front: Occluded front is formed when cold front over•takes warm front and warm air is completely displaced from the ground surface. Cold fronts generally move faster than warm fronts. In fact, the speed of a cold front is about double that of a typical warm front. As a result, a cold front will sometimes overtake an existing warm front. Essentially, an occluded front forms as three air masses meet.

Source: Vajiram & Ravi Yellow book - Concepts of Geography and Natural Resources- Chapter 15: air Mass, Fronts and Cyclones, Page No 130

37. With reference to ‘Plateaus’, consider the following statements: (1) Tibetan plateau is an example of intermont plateau. (2) Many mining areas in the world are located in the plateau regions.

Prelim IAS Test Series (2019) – GS Test 11 (17.02.2019) 28 Geography 1 and Current Affairs Nov 2018

Which of the statements given above is/are correct? (a) 1 only (b) 2 only (c) Both 1 and 2 (d) Neither 1 nor 2

Answer: (c) Explanation: A plateau is a flat, elevated landform that rises sharply above surrounding area on at least one side. Statement (1) is correct: When a plateau is enclosed by fold mountains they are known as intermont plateau. Examples - The Tibetan plateau between the Himalayas and the Kunlun, the Bolivian plateau between two ranges of the Andes. Statement (2) is correct: Plateaus are very important because they are rich in mineral deposits such as iron, copper, gold, diamond, coal, manganese, etc. As a result of these mineral deposits, many mining areas of the world are located in the plateau region. In the Brazilian Plateau, there are huge resources of iron and manganese, particularly in the Minas Gerais area. The Deccan Plateau has deposits of manganese, coal and iron. Source: Vajiram & Ravi Yellow book - Concepts of Geography and Natural Resources- Chapter 8: Major Landforms, Mountains, Plateaus and Plains; Page no 67

38. Which one of the following is a ‘Warm Oceanic Current’? (a) Labrador Current (b) Brazilian Current (c) Peruvian Current (d) Oyashio Current

Answer: (b) Explanation: Labrador Current is found off of the east coast of Canada in the North , and directly affects weather as far away as continental Europe. The current passes along the coasts of Labrador, Newfoundland, and the east coast of Nova Scotia in Canada. The Labrador Current is a cold current with low salinity. Brazil Current is warm current. It runs in the southern direction along the Brazil coast from about 9°S to about 38°S. Its origin begins where the westward flowing South Equatorial Current (SEC) bifurcates (or splits) as it approaches the continental shelf off of Cabo de Sao Roque, Brazil.

Prelim IAS Test Series (2019) – GS Test 11 (17.02.2019) 29 Geography 1 and Current Affairs Nov 2018

Peruvian Current, also called Humboldt Current, is a cold current of the south-east . It is an ocean current that flows along the western coast of South America, affecting the water and air temperatures of coastal Chile and Peru. Oyashio Current is a cold current which originates from the and moves southwards along the coast of the to touch the island of Kurlie. It carries with it the cold water and icebergs from the to the coast of eastern Russia and Japan. Source: Certificate Physical and Human Geography, Goh Cheng Leong, Chapter 12: The Oceans, Page 88-89

39. A traveller crossing the ‘International Date Line’ from west to east ______(a) Gains a day. (b) Loses a day. (c) Neither gains nor loses a day. (d) Initially gains 12 hours but later loses 12 hours.

Answer: (a) Explanation: A traveller crossing the ‘International Date Line’ from west to east gains a day because of the gain in time he encounters. Thus, when it is midnight, Friday on the Asiatic side, by crossing the line eastwards, he gains a day. It will be midnight Thursday on the American side. i.e he experiences the same calendar date twice. Note: A traveller crossing the date line from east to west loses a day because of the loss in time he has made. Source: Vajiram & Ravi Yellow book - Concepts of Geography and Natural Resources, Chapter: About The Earth - Chapter 2, Page No 13

40. Consider the following statements regarding land and sea breeze: (1) Land and sea breezes are infact monsoons on a smaller scale. (2) During the daytime, land breeze is active. Which of the statements given above is/are correct? (a) 1 only (b) 2 only (c) Both 1 and 2 (d) Neither 1 nor 2

Prelim IAS Test Series (2019) – GS Test 11 (17.02.2019) 30 Geography 1 and Current Affairs Nov 2018

Answer: (a) Explanation: Statement 1 is correct: Land and sea breezes, representing a complete cycle of diurnal winds, are in fact, monsoon winds at local scale because they change their direction twice in every 24-hour period. These local diurnal monsoon winds commonly known as land and sea breezes are found in the coastal areas. Statement 2 is incorrect: Sea breeze blows from sea to land, during day time and land breeze moves from land to sea during night due to differential heating and cooling of land and water. During the day time, land gets heated up faster than water. Due to this, the air above the land surface gets heated quickly and rises up thereby developing a low pressure area over the land surface. In comparison to the land, water takes more time to acquire the same temperature as of land. The sea remains comparatively cool which creates a high pressure over it. This pressure difference causes the movement of air from sea towards land. This local wind is called as a Sea Breeze. Thus, sea breeze is most active during daytime. Value addition: At night, land releases the heat quickly thereby lowering the surface temperature and developing a high-pressure area over it. Contrary to land, water releases heat at comparatively lower speed. Thus, sea conserves its heat and remains quite warm due to which a low-pressure area develops over the sea. Because of this pressure difference developed over land and sea, winds move from land towards sea. Such local wind is called as a Land Breeze. Thus, land breeze is active during night. Source: Vajiram & Ravi Yellow book - Concepts of Geography and Natural Resources- Chapter 14: Atmospheric Circulation, Page No 122

41. ‘Nilgiri Hills’ situated in Western Ghats is an example of ______(a) Fold mountain (b) Dome mountain (c) Volcanic mountain (d) Relict mountain

Answer: (d) Explanation: Relict mountains are those mountains which have been eroded by the agents of degradation such as winds, rain, frost and running water. During denudation of an elevated area, some hardened rocks escape from being worn away leaving behind a

Prelim IAS Test Series (2019) – GS Test 11 (17.02.2019) 31 Geography 1 and Current Affairs Nov 2018

Relict Mountain (made up of hard materials) but at the same time the surrounding area gets eroded. Hills like Satpura, Vindhya, Aravali, Nilgiri, Palkonda etc. are the examples of Relict Mountains. Fold Mountains: Such mountains are caused by large scale earth movement when stress is set up in the earth’s crust. The rocks are subjected to compressive forces that produce wrinkling or folding along the line of weakness. Example: Himalayas. Dome Mountains: Such mountains are originated by magmatic intrusions and up-warping of the crustal surface. Dome mountains don’t usually get as high as folded mountains because the force of the magma underneath doesn’t push hard enough. Over a long period, the magma cools to become cold, hard rock. The result is a dome-shaped mountain. Erosion wipes away the outer layers of the mountain, exposing the dome- shaped cooled magma of harder rock. An example of a dome-shaped mountain is Half Dome in the range in California. Volcanic Mountains: These are infact volcanoes which are built up from material ejected from fissures in the earth’s crust. The materials include molten lava, volcanic bombs, ciders, etc. They fall around the vent in successive layers, building up a characteristic volcanic cone. Ex. Mt. Fuji. Source: Vajiram & Ravi Yellow book - Concepts of Geography and Natural Resources- Chapter 8: Major Landforms, Mountains, Plateaus and Plains; Page no 66

42. Consider the following statements about ‘Tornados: (1) Tornados are primarily an over-land phenomenon whereas tropical cyclone is an oceanic phenomenon. (2) In comparison with tropical cyclone, tornados have very small life span. Which of the statements given above is/are correct? (a) 1 only (b) 2 only (c) Both 1 and 2 (d) Neither 1 nor 2

Answer: (c) Explanation: A tornado is a rapidly rotating column of air that is in contact with both the surface of the Earth and a cumulonimbus cloud. The winds of a tornado can reach speeds of up to 480km per hour. They occur more often in late afternoon, when thunderstorms are common, and are more prevalent in spring and summer. Statement 1 is correct. A tornado is an over-land phenomenon. Generally, tornados occur over places where cold and warm fronts converge. On the other hand, tropical cyclones are formed over the oceanic region. They always form over the warm waters of the tropical oceans (sea-surface temperatures must be above 26.5° C) from which they draw their energy. Statement 2 is correct. Tornados have a very small life span and they lasts for minutes/hours. Tropical cyclones last even for days/weeks. They travel thousands of miles, persist over several days, and, during their lifetime, transport significant amounts of heat from the surface to the high altitudes of the tropical atmosphere.

Prelim IAS Test Series (2019) – GS Test 11 (17.02.2019) 32 Geography 1 and Current Affairs Nov 2018

Source: Vajiram & Ravi Yellow book - Concepts of Geography and Natural Resources- Chapter 13: Weather and Climate; Page No.137

43. Which of the following pairs are correctly matched?

Plain Type Example

1. Structural plains : Coromandal plains

2. Plains of glacial erosion : Kashmir

3. Karst Plains : Yugoslavia

Select the correct answer using the code given below: (a) 1 and 2 only (b) 2 and 3 only (c) 3 only (d) 1, 2 and 3

Answer: (d) Explanation: Pair (1) is correct. Structural plains are formed by upliftment or subsidence of land due to diastrophic forces. Here, there is an upliftment of a portion of land beneath the ocean water or submergence of coastal land under ocean water. For example: Great Plains of USA, Coromandal plains of India. Pair (2) is correct. Plains of glacial erosion have rounded peaks and worn down ice scoured shield lands. Example: Kashmir of India, North-Western . Pair (3) is correct. Karst plains have corrugated and undulating surfaces produced by chemical weathering of limestone landscape by water. Example: Karst region of Yugoslavia Source: Vajiram & Ravi Yellow book - Concepts of Geography and Natural Resources- Chapter 8: Major Landforms, Mountains, Plateaus and Plains; Page no 70

44. Consider the following: Assertion (A): Westerlies are stronger in the northern hemisphere than in the southern hemisphere. Reason (R): Northern hemisphere has more land area than the southern hemisphere. Select the correct answer using the code given below: (a) Both Assertion and Reason are true and Reason is the correct explanation of Assertion. (b) Both Assertion and Reason are true but Reason is not the correct explanation of Assertion. (c) Assertion is true but Reason is false. (d) Assertion is false but Reason is true.

Prelim IAS Test Series (2019) – GS Test 11 (17.02.2019) 33 Geography 1 and Current Affairs Nov 2018

Answer: (d) Explanation: The permanent winds blowing from the subtropical high pressure belts (30°- 35° latitude) to the sub-polar low pressure belts (60°- 65° latitude) in both the hemispheres are called westerlies. The general direction of the westerlies is South-West to North-East in the northern hemisphere and North-West to South-East in the southern hemisphere. Because of the dominance of land in the northern hemisphere, the westerlies become more complex and complicated and hence less effective. The westerlies become more vigorous in the southern hemisphere because of lack of land and dominance of oceans. Their velocity increases southward and they become stormy. The velocity of westerlies becomes so great that they are called as roaring forties between the latitude of 40°- 50° S, furious fifties at 50° S and shrieking sixties at 60° S latitude. Source: Certificate Physical and Human Geography, G C Leong, Chapter 4, Page 110

45. A major part of ‘Central Africa’ is forested because (a) Most of the countries in this region are thinly populated. (b) A major part of this region has a rugged topography. (c) The sub-saharan soil is most suitable for forest growth. (d) Equator passes through the middle of the region.

Answer: (d) Explanation: Central Africa lies in equatorial region. Equatorial rainforests forests extend between 5°N and 5°S of the equator. They are found in Congo basin (Central Africa) and of South America. Central African forests represent the second largest area of equatorial rainforest in the world. These forests constitute 35 percent of the African forest area as well as approximately 6 percent of the world forest cover. The Democratic Republic of the Congo contains more than 60 percent of the sub-region's forest area. Value Addition:

Prelim IAS Test Series (2019) – GS Test 11 (17.02.2019) 34 Geography 1 and Current Affairs Nov 2018

According to the UN Food and Agriculture Organization (FAO), indigenous (also known as “old-growth”) forests in Africa are being cut down at a rate of more than 4 mn hectares per year - twice the world’s deforestation average. The Central African Forest Initiative (CAFI), launched at the UN Sustainable Development Summit in September 2015, aims to support governments in the region as they implement reforms and enhance investments to address such challenges as poverty, food insecurity and climate change. These all put pressure on their forests. Source: Certificate Physical and Human Geography, G C Leong, Chapter 15, Page 118

46. Consider the following: Assertion: Jet planes flying in the sky leave a white trail behind them. Reason: The moisture from their engines condenses. Select the correct answer using the code given below: (a) Both Assertion and Reason are true and Reason is the correct explanation of Assertion. (b) Both Assertion and Reason are true but Reason is not the correct explanation of Assertion. (c) Assertion is true but Reason is false. (d) Assertion is false but Reason is true.

Answer: (a) Explanation: Jet planes flying in the sky leave a white trail behind them. These 'white streaks' do have a scientific name – contrails. Flying a plane requires a lot of fuel, which creates a lot of exhaust. Mixed in with that exhaust is water vapour. When planes fly above 25,000 feet or so, the temperature is very cold. When exhaust fumes are shot out of the back of an airplane, the water vapour hits the cold air and condenses, leaving the contrail behind. Thus, the moisture from plane engines condenses. Depending on the humidity, temperature, and the chemical composition of the exhaust, contrails last in the sky for varying amounts of time, and those variables also determine the size and length of the trails. Such trails of condensed moisture can be seen for some time when there is no air movement to disturb them. Source: NCERT Class VII – Chapter: Air, Chapter No. 4, Page no. 26

47. Which of the following statements is/are correct about ‘Earthquakes’? (1) Earthquakes are very common in cratonic zones. (2) Alpide belt represents zone of tectonic plate’s convergence. Select the correct answer using the code given below: (a) 1 only (b) 2 only (c) Both 1 and 2 (d) Neither 1 nor 2

Answer: (b) Explanation:

Prelim IAS Test Series (2019) – GS Test 11 (17.02.2019) 35 Geography 1 and Current Affairs Nov 2018

Statement (1) is incorrect: Cratons are large regions of continental crust that have remained tectonically stable for a prolonged period of time, often a billion years or more. The term ‘craton’ is used to distinguish such regions from mobile geosynclinal troughs, which are linear belts of sediment accumulations subject to subsidence (i.e., downwarping). Cratons are stable part of Earth. Earthquakes are very rare in stable cratonic zones that have hard resistant rocks. Every continent has cratonic areas, notably the core of North America, , , India and most of Australia. They may be covered by a thin shawl of sediment, but often they expose ‘basement’ rocks such as gneiss. Economically they are very important – most of the world’s diamonds come from cratonic areas as do many other valuable deposits. Statement (2) is correct: The Alpide Belt is an orogenic and seismic belt that includes a series of mountain ranges. It refers to a geographic area located in the southern region of Eurasia. This area is prone to earthquake activity and is recorded as having the second highest rate of seismic activity in the world. In fact, the Alpide belt accounts for 15% of all earthquakes worldwide. It was created as the result of tectonic plates pushing into each other and lifting the Earth’s crust upward. The zone stretches as far west as the Atlantic

coast of northern Africa and as as the islands of and . Between these two points, it runs across the Himalayan and Mediterranean regions. A number of major mountain ranges are located in the Alpide belt. The most famous of these ranges include the Himalayas and the Alps. Source: Vajiram & Ravi Yellow book - Concepts of Geography and Natural Resources- Chapter 5: Earthquakes, Page no: 38-39

48. Consider the following statements about asteroids and comets? (1) Both are made up of rocky materials.

Prelim IAS Test Series (2019) – GS Test 11 (17.02.2019) 36 Geography 1 and Current Affairs Nov 2018

(2) Both are mostly found in the ‘Kuiper belt’. Which of the statements given above is/are correct? (a) 1 only (b) 2 only (c) Both 1 and 2 (d) Neither 1 nor 2

Answer: (a) Explanation: Asteroids and comets have a few things in common. Both are celestial bodies orbiting the Sun, and can have unusual orbits, sometimes straying close to Earth or the other planets. Both are composed of leftover materials of solar system. Statement 1 is correct: Asteroids consist of metals and rocky materials. Comets are made up of ice, dust, rocky materials and organic compounds. When comets get closer to the Sun, they lose material because some of their ice melts and vaporizes. Asteroids typically remain solid, even when near the Sun. Statement 2 is incorrect: Asteroids are generally larger chunk of rocks that comes from the asteroid belt located between the orbit of Mars and Jupiter planets. Comets are found in the two main regions of the outer solar system: the Kuiper belt and the Oort cloud. Source: Vajiram & Ravi Yellow book - Concepts of Geography and Natural Resources- Chapter 1: Origin and Evolution of Earth; Page no-4.

49. Steppe grassland witness extremes of temperature. Also, the annual range of temperature is great. Which one of the following statements is the most valid reason behind such extreme temperature patterns? (a) It is a direct result of ‘hot and cold’ local winds. (b) It lies within the torrid zone of the Earth. (c) It is a direct result of continentality. (d) It lies within westerly wind belt.

Answer: (c) Explanation: Eurasian Steppe is temperate grassland. Its location is in the heart of . It experiences steppe climate i.e. temperate continental climate. The annual range of temperature is great (around 60° - 70° C), which is a direct result of continentality. Summers are very warm and winters are very cold in the continental steppes of Eurasia because of the enormous distances from the nearest sea. Though they lie in the westerly wind belt, they are far from maritime influence.

Prelim IAS Test Series (2019) – GS Test 11 (17.02.2019) 37 Geography 1 and Current Affairs Nov 2018

Source: Certificate Physical and Human Geography, G C Leong, Chapter 20, Page 149

50. With reference to the ‘Relief of Oceanic Floor’, which of the following statements is / are not correct? (1) Continental slope region may have underwater canyons. (2) Abyssal plain is normally flat and featureless. Select the correct answer using the code given below: (a) 1 only (b) 2 only (c) Both 1 and 2 (d) Neither 1 nor 2

Answer: (b) Explanation: The ocean floors can be divided into 4 major divisions: Continental Shelf, Continental Slope, Abyssal Plain and Oceanic Deeps. Statement 1 is correct. The most distinctive features of the continental slopes are the submarine canyons. A submarine canyon is a steep-sided valley cut into the seabed of the continental slope, sometimes extending well onto the continental shelf, having nearly vertical walls. These are V-shaped features, often with tributaries, similar to canyons found on dry land. The deepest of the submarine canyons easily rivals similar landforms on the continents. Example: The Monterrey Canyon off the coast of northern California.

Statement 2 is incorrect. At the end of the continental slope and rise, abyssal plain occurs at the deepest portion of the ocean. The average depth of abyssal plain is 3000- 6000 metres and it covers nearly 40 per cent of the ocean. Such plains were once regarded as featureless, but now it has been established that they are as irregular as the continental plains. Source: Vajiram & Ravi Yellow book - Concepts of Geography and Natural Resources - Chapter 17: The Hydrosphere, Page 158-159.

51. With reference to ‘Origin of the Universe’, consider the following statements: (1) Nebula was a large hot and rarefied cloud of gas which rotated slowly. (2) During initial period, sun was surrounded by the dust containing helium only. (3) Weak Nuclear Force formation was one of the results of the big bang. Which of the statements given above is/are correct? (a) 2 only (b) 1 and 3 only (c) 2 and 3 only (d) 1, 2 and 3

Answer: (b) Explanation: Statement 1 is correct. Nebula was a large hot and rarefied cloud of gas and dust. It rotated slowly. Gradually it cooled and contracted and its speed increased. A gaseous ring

Prelim IAS Test Series (2019) – GS Test 11 (17.02.2019) 38 Geography 1 and Current Affairs Nov 2018 was separated from nebula. Later the ring cooled and took the form of planets. The central region of nebula became the sun. This hypothesis was given by Kant and Laplace. Statement 2 is incorrect. The sun was surrounded by dust containing mostly the hydrogen and helium. The hypothesis was given by Otto Schmidt and Carl Weizascar. Statement 3 is correct. One result of the big bang was the formation of the four basic forces in the universe such as: Gravitational force, Weak Nuclear force, Electromagnetic force, Strong Nuclear force. Source: Vajiram & Ravi Yellow book - Concepts of Geography and Natural Resources - Chapter 1: Origin and Evolution of Earth; Page no-1, 2.

52. Consider the following statements: (1) Hotter atmosphere can hold more water that may lead to hurricanes dumping more water on affected areas. (2) Mangkhut typhoon slammed the Philippines recently (3) Florence Hurricane hit the US East coast recently Which of the statement(s) given above is/are correct? (a) 3 only (b) 2 and 3 only (c) 1 and 2 only (d) 1, 2 and 3

Answer: (d) Explanation: Statement 1 is correct. Hotter atmosphere can hold more water that may lead to hurricanes dumping more water on affected areas. Statement 2 is correct. Mangkhut typhoon slammed the Philippines recently. Super Typhoon Mangkhut was a deadly tropical storm wreaking havoc in South East Asia. It devastated the Asian countries. Statement 3 is correct. Florence Hurricane hit the US East coast. Hurricane Florence dumped sheets of rain and flooding the Carolinas in the US.

53. Generally, a cold front but not a warm front, is associated with severe thunderstorms because ______. (a) Warm front completely displaces cold air from the ground surface. (b) Warm front has a steep slope. (c) Cold front completely displaces warm air from the ground surface. (d) Cold front has a steep slope.

Answer: (d) Explanation: Cold fronts are associated with the most violent types of weather such as thunderstorms. Cold front is that sloping frontal surface along which cold air becomes active and aggressive and invades the warm air territory and being denser remains at the ground but forcibly uplifts the warm and light air. Since the air motion is retarded at the ground surface due to friction while the free air above has higher velocity, the cold front becomes much steeper than warm front. This is why the slope of cold front varies from 1:50 to

Prelim IAS Test Series (2019) – GS Test 11 (17.02.2019) 39 Geography 1 and Current Affairs Nov 2018

1:100. Such steep slope encourages vigorous upward motion of warn air ahead of cold front which can lead to the development of intense showers and thunderstorms. On the other hand, the average slope of warm fronts in middle latitudes ranges between 1:100 and 1:400. Thus, the slope of a warm front is much more gradual and the lifting of air is much gentler than commonly observed along a cold front. Consequently, the resultant precipitation is steadier, more wide spread and not very heavy. Clouds and precipitation can be observed at least several hundred kilometres ahead of the surface warm front. Source: Vajiram & Ravi Yellow book - Concepts of Geography and Natural Resources- Chapter 15: Air Mass, Fronts and Cyclones, Page No 130

54. A body of air with horizontally uniform levels of temperature, humidity and pressure is known as: (a) Front (b) Air mass (c) Air zone (d) Pressure cell

Answer: (b) Explanation: An air mass is a large body of air whose properties of temperature and humidity are similar in any horizontal direction. Air masses can cover hundreds of thousands of square miles. Air masses are formed when air stagnates for long periods of time over a uniform surface. The characteristic temperature and moisture of air masses are determined by the surface over which they form. An air mass acquires these attributes through heat and moisture exchanges with the surface. Air masses are classified according to the temperature and moisture characteristics where they develop.  Cold air masses originate in polar regions and are therefore called polar air masses.  Warm air masses usually form in tropical or subtropical regions and are called tropical air masses.  Moist air masses form over oceans and are referred to as maritime air masses.  Dry air masses that form over land surfaces are called continental air masses. Source: Vajiram & Ravi Yellow book - Concepts of Geography and Natural Resources- Chapter 15: Air Mass, Fronts and Cyclones, Page No 129

55. Which of the following was called as a ‘Golden Fibre’? (a) Silk (b) Jute (c) Cotton (d) Wool

Prelim IAS Test Series (2019) – GS Test 11 (17.02.2019) 40 Geography 1 and Current Affairs Nov 2018

Answer: (b) Explanation: Jute was known as the ‘Golden Fibre’ due to its colour and cost-effectiveness. Jute fibre is 100% bio-degradable and recyclable and thus environmentally friendly. Cultivating jute in crop rotations enriches the fertility of the soil for the next crop. Jute also does not generate toxic gases when burnt. India and Bangladesh are the leading producers of jute. Source: NCERT Class VIII – Chapter: Agriculture, Chapter No. 4, Page No 45

56. In context of ‘Mid-Oceanic Ridges’, consider the following statements: (1) They are present at destructive plate margins. (2) Akin to young fold mountains, they achieve great heights. (3) They are geologically inactive. (4) They provide an evidence for sea-floor spreading. Which of the statements given above is/are incorrect? (a) 1, 2 and 3 only (b) 2, 3 and 4 only (c) 1, 2 and 4 only (d) 1, 3 and 4 only

Answer: (a) Explanation: Statement 1 is incorrect. Mid-oceanic ridges are present at constructive plate margins and not destructive plate margins. Statement 2 is incorrect. Mid-oceanic ridges are a system of lofty underwater mountain system. They achieve great heights. These ridges can also reach as high as 3,000 feet above the seafloor. However, their formation, age and height differ greatly when compared to young fold mountain systems of the world. Statement 3 is incorrect. Mid ocean ridges are geologically active. This means that they experience volcanic eruptions as well as earthquakes. Statement 4 is correct. Paleomagnetic rocks on either side of the mid oceanic ridges provide the most important evidence to the concept of Sea Floor Spreading. Hess propounded that the topography of the seafloor could be explained if the seafloor moves sideways, away from the oceanic ridges. Hess postulated that magma rose from the interior of the Earth and formed new oceanic crust along the mid-ocean ridges. This hypothesis was tested using paleomagnetism. When lava is extruded at any mid-ocean ridge, the rock it forms becomes magnetized and acquires the magnetic polarity that exists at the time the lava cools. As the crust moves away from mid-ocean ridges, it contains a continuous record of the Earth's changing magnetic polarity. Source: Vajiram & Ravi Yellow book - Concepts of Geography and Natural Resources- Chapter 17: The Hydrosphere, Page 156-161.

57. With reference to ‘Tsunami’, consider the following statements:

Prelim IAS Test Series (2019) – GS Test 11 (17.02.2019) 41 Geography 1 and Current Affairs Nov 2018

(1) It is triggered due to seismic activity only. (2) It is difficult to detect in open ocean waters. Which of the statements given above is/are incorrect? (a) 1 only (b) 2 only (c) Both 1 and 2 (d) Neither 1 nor 2

Answer: (a) Explanation: The word tsunami is a Japanese word, represented by two characters: tsu, meaning, ‘harbour’, and nami meaning, ‘wave’. Tsunami is a set of ocean waves caused by any large, abrupt disturbance on the sea- surface. Statement 1 is incorrect. When a sudden displacement of a large volume of water occurs, or if the sea floor is suddenly raised or dropped, big tsunami waves can be formed. It can originate from a wide gamut of factors like earthquake, submarine volcanic eruptions, submarine landslides and meteorite impacts. However, powerful undersea earthquakes are responsible for most tsunamis. Statement 2 is correct. In the deep ocean, tsunami wave amplitude is usually less than 1 m. The crests of tsunami waves may be more than a hundred kilometres or more away from each other. Therefore, passengers on boats at sea, far away from shore where the water is deep, will not feel nor see the tsunami waves as they pass by underneath at high speeds. The tsunami may be perceived as nothing more than a gentle rise and fall of the sea surface. That’s why it is difficult to detect them in open ocean waters. Value Addition: Tsunami travels at a speed that is related to the water depth. As a tsunami leaves the deep water of the open ocean and travels into the shallower water near the coast, the water depth decreases and the tsunami slows down. The tsunami's energy flux, which is dependent on both its wave speed and wave height, remains nearly constant. Consequently, as the tsunami's speed diminishes as it travels into shallower water, its height grows. Because of this effect, a tsunami, imperceptible at sea, may grow to be several meters or more in height near the coast. Source: Vajiram & Ravi Yellow book - Concepts of Geography and Natural Resources- Chapter 17: The Hydrosphere, Page 172.

58. With reference to ‘Oceanic Currents’, which of the following statements is/are correct? (1) Oceanic currents flow clockwise in northern hemisphere and anti-clockwise in southern hemisphere. (2) Warm ocean currents are mainly responsible for high salinity in of Palestine. Select the correct answer using the code given below: (a) 1 only (b) 2 only (c) Both 1 and 2 (d) Neither 1 nor 2

Prelim IAS Test Series (2019) – GS Test 11 (17.02.2019) 42 Geography 1 and Current Affairs Nov 2018

Answer: (a) Explanation: An ocean current is any more or less permanent or continuous, directed movement of ocean water. Ocean currents drive oceanic water for thousands of kilometres away. Ocean currents while flowing along the coasts modify their weather conditions in a number of ways. Statement 1 is correct. The earth’s rotation deflects freely moving objects, including ocean currents to the right. In the northern hemisphere this is a clockwise direction. For example: the circulation of the Gulf Stream Drift and the Canaries Current. In the southern hemisphere, it is anti-clockwise direction. For example: the Brazilian Current and the West Wind Drift. Statement 2 is incorrect. In the open oceans where ocean currents freely flow, salinity tends to be near the average 35 parts per thousand or even little lower. The range of salinity is negligible where there is free mixing of water by surface and sub-surface currents. But in enclosed seas, which are areas of inland drainage, such as Dead Sea; waters do not mix freely with the ocean water and they are not penetrated by ocean currents. Hence, the salinity is very high. Salinity in the Dead Sea is around 250 parts per thousand. Source: Vajiram & Ravi Yellow book - Concepts of Geography and Natural Resources- Chapter 17: The Hydrosphere, Page 167-168.

59. Consider the following pairs:

Topography Type Landform

1. Fluvial : Truncated spurs

2. Glacial : Interlocking spurs

3. Arid : Loess deposits

4. Coastal : Sink hole

Which of the pairs given above is/are correctly matched? (a) 2 only (b) 2 and 4 only (c) 3 only (d) 1, 2 and 3 only

Answer: (c) Explanation:

Prelim IAS Test Series (2019) – GS Test 11 (17.02.2019) 43 Geography 1 and Current Affairs Nov 2018

Interlocking spurs are related to Fluvial topography. They are features found in the upper reaches of river valleys. They are erosional features, meaning that they are formed by water flowing over the land and eroding it as it moves Truncated spurs are blunt-ended rock ridges which descend from the steep sides of a U-shaped valley or glacial trough. They are often separated by hanging valleys. Due to high erosion and load, glacial topography has such truncated spurs. Arid topography results in carrying away of fine dust particles. Such particles get deposited in far-away locations from the source of origin forming loess deposits. Karst is an area of land made up of limestone. Limestone, also known as chalk or calcium carbonate, is a soft rock. Because carbonate rocks can be dissolved by groundwater, karst areas are often characterized by sinkholes. Only, 3rd pair is correctly

matched. Source: Vajiram & Ravi Yellow book - Concepts of Geography and Natural Resources- Chapter 9: Erosional and Depositional Landforms, Page 73, 75, 76.

60. On a clear January night, a direct flight - Delta 163 started its journey from Toronto towards Dhaka. After crossing Atlantic Ocean, the aeroplane encountered heavy torrential showers and had to make an emergency landing at an airport X. The flight - Delta 163 began its remaining journey after a few hours. When it crossed Kandahar city, once again bad weather conditions were witnessed. Though this time, the aeroplane continued the remaining journey for 2 more hours, it was

Prelim IAS Test Series (2019) – GS Test 11 (17.02.2019) 44 Geography 1 and Current Affairs Nov 2018

finally forced to make an emergency landing at an airport Y due to rain and poor visibility. It is most likely that the two airports X and Y were ______and ______respectively. (a) Paris, Mumbai (b) Chicago, Delhi (c) Athens, Ahmadabad (d) Rome, Chandigarh

Answer: (d) Explanation: In Northern Hemisphere, during January month, rainfall is most probable in Mediterranean region and Northern India. Rainfall in winter is a unique feature of Mediterranean climate. Also, ‘Western Disturbances’ originate over the east Mediterranean Sea and travel eastwards across West Asia, Iran-Afghanistan and Pakistan before they reach the north- western parts of India. On their way, they pick up moisture from the in the north and the Persian Gulf in the South. The western disturbances cause rainfall in the northern India and heavy snowfall in the western Himalayas. From the given options Delhi and Chandigarh are the places which normally come under the influence of western disturbances. But Chicago lies on west of Atlantic Ocean whereas Rome experiences Mediterranean climate. Hence, the most probable airports represented by X and Y are Rome and Chandigarh respectively. Source: Vajiram & Ravi Yellow book - Concepts of Geography and Natural Resources- Chapter 16: World Climatic Regions, Page 141-142.

61. Which among the following climate types are conspicuously absent in the southern hemisphere? (1) Mediterranean Climate (2) Siberian Climate (3) Laurentian Climate (4) British Climate Select the correct answer using the code given below: (a) 1 and 2 only (b) 2 and 3 only (c) 2 and 4 only (d) 2, 3 and 4

Answer: (b) Explanation: Siberian Climate is conspicuously absent in the southern hemisphere because of the narrowness of the southern continents in the high latitudes. Siberian climate is experienced only in the northern hemisphere where the continents within the high latitudes have a broad east-west spread. Laurentian Climate is absent in southern hemisphere because only a small section of southern continents extends south of the latitude of 40° S. Note: Mediterranean Climate is preset in southern Australia, south western tip of Africa and central Chile in southern hemisphere.

Prelim IAS Test Series (2019) – GS Test 11 (17.02.2019) 45 Geography 1 and Current Affairs Nov 2018

British Climate is present in southern Chile and most parts of New Zealand in southern hemisphere. Source: Certificate Physical and Human Geography, G C Leong, Chapter 19 - 24, Page 141 - 177

62. Consider the following: Assertion: Equatorial zone has low diurnal and annual range of temperature. Reason: High temperature and high humidity cause orographic rain in most afternoons in the equatorial zone. Select the correct answer using the code given below: (a) Both Assertion and Reason are true and Reason is the correct explanation of Assertion. (b) Both Assertion and Reason are true but Reason is not the correct explanation of Assertion. (c) Assertion is true but Reason is false. (d) Assertion is false but Reason is true.

Answer: (c) Explanation: The equatorial climate is characterized by two major properties e.g.: 1) Uniformly high temperature throughout the year: As mid-day sun is almost overhead throughout the year and there is little difference between the lengths of day and night during the year. Cloudiness and heavy precipitation help to moderate the daily temperature. In addition, regular land and sea breeze assist in maintaining a truely equable climate. Usually, mid-day temperature rises to 29°C-34°C and comes down to 21°C-24°C during nights. So, the diurnal range of temperature is small. The equatorial region receives maximum amount of insolation which causes uniformly high temperature throughout the year. There is no winter season. Hence, annual range of temperature is very low. The mean annual range of temperature is between 0.5°C and 3°C. Hence, ‘Assertion’ statement is correct. 2) Uniformly adequate rainfall throughout the year received by convective mechanism: A strong daily vertical convective mechanism is common in equatorial regions because of intense heating of ground surface caused by high amount of insolation. Such convective mechanism creates thunderstorms, which yield heavy rainfall on daily basis throughout the equatorial regions. Thus, high temperature and high humidity cause is convective rainfall in nature. Hence, ‘Reason’ statement is incorrect. Source: Vajiram & Ravi Yellow book - Concepts of Geography and Natural Resources- Chapter 16: World Climatic Regions, Page 141-142.

63. With reference to ‘Fluvial Erosion and Deposition’, consider the following statements: (1) Lateral erosion dominates during the upper course of a river. (2) Vertical down cutting results in U-shaped valley which is found in the Himalayas. (3) Both erosion and deposition are active in an Oxbow lake formation. (4) Arcuate deltas are very rarely formed by rivers.

Prelim IAS Test Series (2019) – GS Test 11 (17.02.2019) 46 Geography 1 and Current Affairs Nov 2018

Which of the statements given above is/are incorrect? (a) 1, 2 and 3 only (b) 1, 2 and 4 only (c) 2, 3 and 4 only (d) 1, 3 and 4 only

Answer: (b) Explanation: Statement 1 is incorrect. During the young stage of a river i.e. in the upper course, vertical down cutting dominates

over lateral erosion. Statement 2 is incorrect. Vertical downcutting results in V-shaped valley found in Himalayas. Statement 3 is correct. Both erosion and deposition are active in an Ox-bow lake formation as shown in the figure. Statement 4 is incorrect. Most of the river deltas in the world are arcuate deltas. They have rounded shape, lots of distributaries and maximum deposition is towards centre. Source: Vajiram & Ravi Yellow book - Concepts of Geography and Natural Resources- Chapter 10: The Drainage System and Landforms, Page 81-83.

64. The advent and withdrawal of Monsoon in India results in a peculiar change in , reflected by which of the following statements given below? (a) Indian Ocean experiences reduction in ocean water temperature on western coastal margins. (b) Bay of Bengal and Arabian Sea experience a very high variation in salinity. (c) Westerlies strengthen over the Indian Ocean. (d) Indian Ocean experiences change in direction of oceanic currents.

Answer: (d) Explanation:

Prelim IAS Test Series (2019) – GS Test 11 (17.02.2019) 47 Geography 1 and Current Affairs Nov 2018

The Indian Ocean gyre, located in the Indian Ocean, is one of the eighteen major oceanic gyres i.e large systems of rotating ocean currents. Normally moving counter-clockwise in the winter, the Indian Ocean gyre reverses direction with the arrival of the seasonal winds of the south-west monsoon. In the summer, the land is warmer than the ocean, so surface winds blow from the ocean to the land. However, during the winter, these temperatures reverse, making the winds blow from the land to the ocean. Due to this seasonal wind cycle, the currents of the Indian Ocean, which make up the Indian Ocean gyre, are directly affected and experience reversal in their direction.

Source: Certificate Physical and Human Geography, G C Leong, Chapter 12:The Oceans, Page 90

65. With respect to ‘Climatic Regions of the World’, which of the following statements is / are not correct? (1) Temperate grasslands differ from tropical grassland as there are more trees marking the landscape in temperate grasslands. (2) Steppe type of climate is characterized by an alternate hot, rainy season and cool, dry season. (3) Chaparral shrublands are typical to Mediterranean climate. Select the correct answer using the code given below: (a) 1 only (b) 1 and 2 only (c) 2 and 3 only (d) 1, 2 and 3

Answer: (b) Explanation: Statement 1 is incorrect. Temperate grasslands (steppe) are treeless, and have short grass, due to continentality (i.e. they are deep within continents where rain bearing winds hardly reach). Note: Tropical grasslands (Savanna) are marked by tall grass and short trees.

Prelim IAS Test Series (2019) – GS Test 11 (17.02.2019) 48 Geography 1 and Current Affairs Nov 2018

Statement 2 is incorrect. Sudan i.e Savanna type of climate is characterized by an alternate hot, rainy season and cool, dry season. The length of the wet and dry seasons differs with the locality. Statement 3 is correct. Mediterranean climate is marked by summer dry-winter wet setting which is caused by seasonal shifting of wind belts. It results in short shrubs marking the landscape with waxy and/or thorny leaves. Also, due to man’s interference the original woodlands have degenerated to scrub vegetation with scattered, stunted tress ad tall bushes. Such vegetation is known as Chaparral. It is a shrub land, and not grassland. Source: Certificate Physical and Human Geography, G C Leong, Chapter 15-17, Page 116-130

66. Which among the following international treaty prohibits trade in wild animals and plants? (a) CITES (b) Helsinki Declaration (c) Convention on Biological Diversity (d) Vienna Convention

Answer: (a) Explanation: The United Nations’ Convention on International Trade in Endangered Species of Wild Fauna and Flora (CITES) is a treaty among 183 nations. CITES is designed to eliminate wildlife trafficking and ensure legal international trade in wild animals and plants does not threaten their survival. It was founded as a conservation convention to restrict and even prohibit any international trade in wildlife unless it is proven to cause no risk for the species involved. Its Appendix I includes all species threatened with extinction which are or may be affected by trade. Trade in specimens of these species are subjected to particularly strict regulation in order not to endanger further their survival and is only authorized in exceptional circumstances. The Declaration of Helsinki is a set of ethical principles regarding human experimentation developed for the medical community by the World Medical Association. The Convention on Biological Diversity (CBD) founded in 1992 at the Rio Summit, is the legally binding agreement on the use and conservation of biological diversity. The Vienna Convention for the Protection of the Ozone Layer is a multilateral environmental agreement signed in 1985 that provided frameworks for international reductions in the production of chlorofluorocarbons due to their contribution to the destruction of the ozone layer and resulting increase in the threat of skin cancer. Source: NCERT Class VIII – Chapter: Land, Soil, Water, Natural Vegetation and Wildlife Resources, Chapter no. 2:, Page no. 21

67. Consider the following statements regarding ‘Gas Hydrates’: (1) These are crystalline forms of methane and water. (2) In India, their reserves are found in the deep sea off the Andhra Pradesh coast. Which of the statements given above is/are correct? (a) 1 only

Prelim IAS Test Series (2019) – GS Test 11 (17.02.2019) 49 Geography 1 and Current Affairs Nov 2018

(b) 2 only (c) Both 1 and 2 (d) Neither 1 nor 2

Answer: (c) Explanation: Gas hydrates are a crystalline solid that looks and acts much like ice, but it contains huge amounts of methane. It is known to occur on every continent; and it exists in huge quantities in marine sediments in a layer several hundred meters thick directly below the sea floor. It is not stable at normal sea-level pressures and temperatures. Statement (1) is correct: Gas hydrates are crystalline form of methane and water. Statement (2) is correct: ONGC has struck gas hydrate reserves in the deep sea off the Andhra Pradesh coast. The reserves are located in the Krishna-Godavari basin, which came into the limelight about a decade ago and the fresh reserves are estimated to be around 134 trillion cubic feet. Value Addition: Gas hydrates exist in shallow sediments of outer continental margins. They are envisaged as a viable major energy resource for future. Thus, delineation of gas hydrates by geophysical methods is very important for evaluating the resource potential along the Indian continent margin with a view to meet the overwhelming demand of energy for India. Source: (1) Vajiram & Ravi Yellow book - Concepts of Geography and Natural Resources - Chapter 1: Natural Resources; Page no. 192 (2) https://www.ongcindia.com/wps/wcm/connect/en/about-ongc/new-initiatives-in- energy/gas-hydrates/

68. The Pole star helps us know about the: (1) Direction of a place (2) Latitude of a place Which of the statements given above is/are correct? (a) 1 only (b) 2 only (c) Both 1 and 2 (d) Neither 1 nor 2

Answer: (c) Explanation: Pole star also called as North Star indicates the north direction. It always remains in the same direction and is located with the help of Saptarishi constellation. By measuring the angle of a pole star from one’s place, we can know the latitude of a place. Source: NCERT Class VI, Chapter 2: Globe – Latitudes and Longitudes, Page 11

69. Lithification is a process of (a) Soil formation (b) Water percolation (c) Change in form of rocks (d) Rock formation

Prelim IAS Test Series (2019) – GS Test 11 (17.02.2019) 50 Geography 1 and Current Affairs Nov 2018

Answer: (d) Explanation: Lithification is the process that turns loose, unconsolidated sediment into solid sedimentary rock. Lithification occurs by two ways: compaction and cementation. Compaction is the consolidation of sediments due to the intense pressing weight of overlying deposits. Compaction happens when sediments get buried. This literally squishes the sediment grains together, compressing them into a mass. With sufficient pressure and the passage of time, the grains get rearranged and more organized. As the sediments consolidate, the original pore space that divided them is reduced and any water that was in those spaces is squeezed out. Cementation is where new minerals stick the grains together – just as cement binds sand grains in a bricklayer’s mortar. If you look carefully at the microscope photo, you can see mineral crystals that have grown around the sediment grains and bonded them together. Common cementing minerals are calcite (CaCO3), silica (SiO2), iron oxides and clay minerals. Note: The process of soil formation is called as pedogenesis whereas change in form of rocks is called as metamorphosis. Source: NCERT Class 11, chapter 5: Minerals and Rocks, page 42

70. Consider the following statements about ‘landslides’: (1) Gravity is the primary driving force for the occurrence of landslide. (2) Recently, a real-time landslide warning system has been set up in the Sikkim- Darjeeling belt of north-eastern Himalayas. Which of the statements given above is/are correct? (a) 1 only (b) 2 only (c) Both 1 and 2 (d) Neither 1 nor 2

Answer: (c) Explanation: Statement 1 is correct. Landslide, a gravity-led phenomenon, occurs when the slope changes from a stable to an unstable condition. A change in the stability of a slope can be caused by a number of factors, acting together or alone. Natural causes of landslides include: Heavy rain, Earthquake, Volcanic eruptions, etc. Landslides are aggravated by

Prelim IAS Test Series (2019) – GS Test 11 (17.02.2019) 51 Geography 1 and Current Affairs Nov 2018 human activities, such as: Deforestation, cultivation and construction activities, vibrations from machinery or traffic, blasting, etc Statement 2 is correct. Recently, a real-time landslide warning system has been set up in the Sikkim-Darjeeling belt of north-eastern Himalayas, which is highly vulnerable to landslides. The warning system consists of over 200 sensors that can measure geophysical and hydrological parameters like rainfall, pore pressure and seismic activities. Source – NCERT class 11, chapter 6: Geomorphic Process, page 53

71. Which of the following geographical conditions makes hunting and fishing the principal means of livelihood in the Tundra region? (1) Sparse population (2) Infertile dissected terrain (3) Harsh climate (4) Scarcity of vegetation Select the correct answer using the code given below: (a) 1 and 2 only (b) 2 and 3 only (c) 1 and 3 only (d) 2, 3 and 4 only

Answer: (d) Explanation: Human behaviour in the Arctic Tundra is largely controlled by environment. The Arctic region provides a unique example in which the basic needs of the people; socio-economic and cultural aspects of life are influenced closely by the environmental factors. The people in the Tundra depend on hunting and fishing for their sustenance due to adverse climatic conditions. Infertile dissected terrain and scarcity of vegetation are also the factors affecting the hunting and fishing as principal means of livelihood. Source: Certificate Physical and Human Geography, G C Leong, Chapter 25: The Arctic or Polar Climate, Page 185

72. Match List-I with List-II and select the correct answer using the code given below:

List-I List-II (Continent) (Deserts) A. Asia 1. Atacama B. Africa 2. Mojave C. North America 3. Kalahari D. South America 4. Gobi

Codes A B C D (a) 3 2 1 4 (b) 3 1 4 2 (c) 4 2 1 3 (d) 4 3 2 1

Prelim IAS Test Series (2019) – GS Test 11 (17.02.2019) 52 Geography 1 and Current Affairs Nov 2018

Answer: (d) Explanation:

Gobi desert: Situated to the south-east of Mongolian plateau and extends into China, Asia. Kalahari: Semi desert region of Botswana. It is home to one of Africa’s oldest races – the Kalahari Bushmen. Mojave: Situated in south-western part of North America. Atacama: A coastal desert in the northern Chile and southern Peru in South America. It is world’s driest desert. Source: Vajiram & Ravi Yellow book - Concepts of Geography and Natural Resources - Chapter 16: World Climatic Regions; Page no. 145

73. Which among the following is called as ‘Black Gold’? (a) Coal (b) Graphite (c) Petroleum (d) Whale oil

Answer: (c) Explanation: Black gold is a term applied to oil or petroleum, which is black when it comes out of the ground and is worth a great deal of money. Prior to the mid-19th century, oil from animal tallow (fat) and whale blubber was used as a lubricant and fuel. Source: NCERT – Class VIII – Chapter: Mineral and Power Resources, Chapter no. 3, Page no. 31

Prelim IAS Test Series (2019) – GS Test 11 (17.02.2019) 53 Geography 1 and Current Affairs Nov 2018

74. experiences ‘Western Disturbances’ during winter season. Such disturbances originate from ______and end in ______? (a) Sea, Philippine Sea. (b) Caspian Sea, South China Sea. (c) Mediterranean Sea, East China Sea. (d) East Sea, Mediterranean Sea.

Answer: (c) Explanation: Western Disturbance is the name given to ‘Temperate Cyclones’, which originate over Mediterranean Sea. It is a low pressure area or a trough over surface or the upper-air in the westerly winds regime, north of 20°N. It causes changes in pressure, wind pattern and temperature fields. It is accompanied by cloudiness, with or without precipitation. The term Western Disturbance (WD) was coined by Indian meteorologists for describing the systems moving from the west to east direction. Western Disturbances form due to interaction of cold-dry polar air mass and warm-humid tropical air mass. The cyclone such formed due to interaction of warm and cold fronts moves eastwards, under the influence of westerlies. It travels to India via Iran, Afghanistan, Pakistan and moves onwards to China, where it finally reaches East China Sea and dissipates. Source: Certificate Physical and Human Geography, G C Leong, Chapter 14, Page 108

75. The oceanic currents have a great influence on climate and marine ecosystem. In context of recently discovered oceanic current in Indian Ocean, consider the following statements: (1) It is a cold oceanic current which flows towards equator. (2) It flows along south-west coast of Madagascar. Which of the statements given above is/are correct? (a) 1 only (b) 2 only (c) Both 1 and 2 (d) Neither 1 nor 2

Answer: (b) Explanation: South African and French researchers have discovered the existence of a coastal current off Madagascar in Indian Ocean. The newly- described current is called as the South-west Madagascar Coastal Current.

Prelim IAS Test Series (2019) – GS Test 11 (17.02.2019) 54 Geography 1 and Current Affairs Nov 2018

Statement 1 is incorrect. It is a warm oceanic current flowing pole ward in the southern hemisphere. Statement 2 is correct. It flows along south-west coast of Madagascar. Source: (1) https://www.csir.co.za/discovery-new-coastal-current-madagascar (2) https://www.news24.com/Green/News/discovery-of-new-ocean-current-personal-for- csir-researcher-20180409

76. The first human-made object to touch down on the moon's surface was ______and it belonged to ______. (a) Pioneer 1, United States (b) Chang'e 1, China (c) Luna-2, Soviet Union (d) Apollo 11, United States

Answer: (c) Explanation:  Pioneer 1: On October 11, 1958, Pioneer 1 became the first spacecraft under the auspices of NASA, the newly formed space agency of the United States.  Chang'e 1: It was an unmanned Chinese lunar-orbiting spacecraft, part of the first phase of the Chinese Lunar Exploration Program. The spacecraft was named after the Chinese Moon goddess, Chang'e.  Luna 2 or Lunik 2: It was the second of the Soviet Union's Luna programme spacecraft launched to the Moon. It was the first spacecraft to reach the surface of the Moon, and the first man-made object to land on another celestial body. On September 14, 1959, it hit the Moon's surface east of Mare Imbrium near the craters Aristides, Archimedes, and Autolycus.  Apollo 11: Apollo 11 was the spaceflight that landed the first two people on the Moon in July 1969. Commander Neil Armstrong and lunar module pilot Buzz Aldrin, both American, landed the Apollo Lunar Module Eagle on Moon.

77. Which among the following infections/diseases have scientifically been proven to cause birth defects? (1) Zika Fever (2) Malaria Fever (3) Rubella Infection (4) Dengue Fever Select the correct answer using the code given below: (a) 1 and 2 only (b) 1 and 3 only (c) 1, 2 and 3 only (d) 2, 3 and 4 only

Answer: (b) Explanation: Zika Fever: Microcephaly is a birth defect in which a baby’s head is smaller than expected when compared to babies of the same sex and age. Babies with microcephaly often have smaller brains that might not have developed properly. Zika virus infection

Prelim IAS Test Series (2019) – GS Test 11 (17.02.2019) 55 Geography 1 and Current Affairs Nov 2018 during pregnancy is a cause of microcephaly. During pregnancy, a baby’s head grows because the baby’s brain grows. Microcephaly can occur because a baby’s brain has not developed properly during pregnancy or has stopped growing after birth. Rubella Infection: Infection with the rubella virus during pregnancy is very serious. Common complications include miscarriage, premature delivery, and death of the foetus. In those babies who are born alive, a condition called congenital rubella syndrome can result. Congenital rubella syndrome leads to eye, ear, and heart defects as well as microcephaly, autism, and mental and motor delay. These issues are permanent.

78. Recently some sites have been accorded ‘in principle’ approval by Government of India to set up nuclear reactors in future. In this context, which of the following pairs are correctly matched?

Nuclear Plant Site State 1. Jaitapur : Maharashtra 2. Haripur : Madhya Pradesh 3. Bhimpur : West Bengal

Select the correct answer using the code given below: (a) 1 only (b) 1 and 2 only (c) 1 and 3 only (d) 2 and 3 only

Answer: (a) Explanation: Recently, some sites have been accorded ‘in principle’ approval by Government of India to set up nuclear reactors in future.

State Site In Cooperation with

Maharashtra Jaitapur France 6 X 1650 MW

Andhra Pradesh Kovvada United States of America 6 X 1208 MW

Gujarat Chhaya Mithi Virdi United States of America 6 X 1000

West Bengal Haripur Russian Federation 6 X 1000

Madhya Pradesh Bhimpur Indigenous PHWR 4 X 700

Prelim IAS Test Series (2019) – GS Test 11 (17.02.2019) 56 Geography 1 and Current Affairs Nov 2018

79. The Ministry of Earth Sciences (MoES) of India has commissioned two very high resolution (12 km grid scale) state-of-the-art global Ensemble Prediction Systems (EPS) for generating probabilistic predictions of ______. (a) Weather (b) Tsunami (c) Earthquake (d) Landslide

Answer: (a) Explanation: The Ministry of Earth Sciences (MoES) has commissioned two very high resolution (12 km grid scale) state-of-the-art global Ensemble Prediction Systems (EPS) for generating operational 10-days probabilistic forecasts of weather. The EPS will enhance the weather information being provided by the current models by quantifying the uncertainties in the weather forecasts and generate probabilistic forecasts. The probabilistic forecasts of severe weather events at 12 km grid scale across India will greatly help the disaster management authorities and other users in making better emergency response decisions by explicitly accounting for the uncertainty in weather forecasts.

80. The term – ‘Von Kramer’ was in news recently. It refers to ______(a) World’s first artificial intelligence news anchor. (b) Family of six industry-standard microprocessors. (c) Crater within the South Pole basin on the Moon. (d) Polarimetry doppler weather radar.

Answer: (c) Explanation: On Moon, China's Chang’e4 landed on the Von Kramer crater within the South Pole- Aitken basin, known to be the biggest depression in the solar system. Studying the composition of ancient craters such as the Von Kramer can help scientists gain an insight into the asteroids that rained down on the Earth during the planet’s youth. Understanding the history of contact with these heavenly bodies could yield important clues to the origins of life on the earth.

81. When performance of India and Bangladesh are compared then Bangladesh holds a better position than India on which among the following socio-economic indicators: (1) Life Expectancy at Birth (2) Infant Mortality Rate (3) Per Capita Income (4) Total Fertility Rate Select the correct answer using the code given below: (a) 1 and 2 only (b) 1 and 4 only (c) 1, 2 and 4 only (d) 2, 3 and 4 only

Answer: (c)

Prelim IAS Test Series (2019) – GS Test 11 (17.02.2019) 57 Geography 1 and Current Affairs Nov 2018

Explanation: After trailing its big neighbour for four decades, Bangladesh has gone ahead of India on certain socio-economic indicators.

82. Which of the following pairs are correctly matched?

Report Published by

1. Global Status Report (Renewables) : REN21

2. Global Financial Stability Report : World Economic Forum

3. Global Money Laundering Report : World Bank

Select the correct answer using the code given below: (a) 1 only (b) 1 and 2 only (c) 1 and 3 only (d) 2 and 3 only

Answer: (a) Explanation:

Prelim IAS Test Series (2019) – GS Test 11 (17.02.2019) 58 Geography 1 and Current Affairs Nov 2018

Pair 1 is correct. Global Status Report (Renewables) is published by REN21. REN21 is the global renewable energy policy network that connects a wide range of key actors. REN21’s goal is to facilitate knowledge exchange, policy development and joint action towards a rapid global transition to renewable energy. REN21 facilitates the collection of comprehensive and timely information on renewable energy. Pair 2 is incorrect. The Global Financial Stability Report (GFSR) is a semi-annual report by the International Monetary Fund (IMF) that assesses the stability of global financial markets and emerging-market financing. Pair 3 is incorrect. The Global Money Laundering (ML) and Terrorist Financing (TF) Threat Assessment (GTA) report provides an assessment of the global systemic ML/TF threats. The document prepared by Financial Action Task Force on Money Laundering is aimed at raising the level of understanding of these threats and their negative impact, and help governments to take decisive action to minimise the harms they can cause.

83. Recently, ‘Digital Economy’ Meeting was held in Salta, Argentina to promote policies and actions that catalyze digital transformations. This meeting was held under the aegis of (a) G20 (b) BRICS (c) ASEAN (d) SAARC

Answer: (a) Explanation: At the G20 Digital Economy Ministerial Meeting in Salta, in the north-west of Argentina, G20 ministers and senior officials issued a declaration that reflects the G20’s commitment towards promoting “policies and actions that catalyze digital transformations. Building on the contributions of the G20 Digital Economy Task Force, focus was on efforts to create conditions that help governments, the private sector and civil society; maximize the benefits and confront the challenges posed by technological progress. The other focus area of the meeting was digital inclusion, in particular the gender divide.

84. The Union Ministry of Finance has advised banks in India to consider issuing contactless debit cards from end of 2018. Which of the following are the features of a contactless debit cards? (1) The range at which such card can be read is very large. (2) They have an embedded antenna. (3) They work on infrared technology. Select the correct answer using the code given below: (a) 1 only (b) 2 only (c) 1 and 2 only (d) 1, 2 and 3

Answer: (b) Explanation:

Prelim IAS Test Series (2019) – GS Test 11 (17.02.2019) 59 Geography 1 and Current Affairs Nov 2018

By holding a contactless card just one to two inches away from the terminal, consumers can quickly and easily pay for goods and services. Point (1) is incorrect. The range at which a card can be read is very small The maximum range a contactless card communicates at is 4 centimeters. Point (2) is correct. There is an antenna in the contactless card, which transmits encrypted (256-bit) data. It is protected via a digital signature, making it secure. Point (3) is incorrect. Contactless cards use NFC - Near field communication (NFC). NFC-enabled cards are issued by banks that allow users to just tap it against a compatible PoS machines instead of having to swipe it, which requires physical contact. But the near field communication (NFC) technology in contactless cards uses a 13.56Mhz radio frequency technology that only transmits digital data within a very short range (typically 4 cm or less).

85. Recently, Sevastopol was in news. It is ______. (a) India’s new research station built near North Pole to study negative effects of climate change. (b) A campaign launched by election commission of India in support of temper proof voting machines. (c) The procedure followed by the members of Sri-Lankan Parliament to elect their new Prime Minister. (d) A warm water naval port in Ukraine which Russia can use for next few years.

Answer: (d) Explanation: Through 1997 treaty, Russia was given the right by Ukraine to use the port of Sevastopol for 20 years. Sevastopol is the only warm water base that Russia has access to in the strategically important region. It has a great geopolitical significance for Russia. In 2009, Ukraine sent out signals that the treaty would not be extended when it expires in 2017. Finally, an agreement was made in 2010 to extend the treaty by 25 years in 2017 with a provision for an additional 5 years taking it to 2047.

Prelim IAS Test Series (2019) – GS Test 11 (17.02.2019) 60 Geography 1 and Current Affairs Nov 2018

86. Hypoxia, or oxygen depletion, is an environmental phenomenon where the concentration of dissolved oxygen in the water column decreases to a level that can no longer support living aquatic organisms. Hypoxia can be linked to ______. (1) Lead (2) Nitrogen (3) Phosphorus (4) Arsenic Select the correct answer using the code given below: (a) 1 and 2 only (b) 1 and 3 only (c) 2 and 3 only (d) 2, 3 and 4 only

Answer: (c) Explanation: The largest human-caused hypoxic zone currently affecting the United States, and the second largest worldwide, occurs in the northern Gulf of Mexico adjacent to the Mississippi River. The hypoxic zone in the northern Gulf of Mexico is an area where water near the bottom of the Gulf contains less than two parts per million of dissolved oxygen. Major events leading to the formation of hypoxia in the Gulf of Mexico include:  Freshwater discharge and nutrient loading mostly nitrogen and phosphorous  Nutrient-enhanced primary production, or eutrophication  Decomposition of biomass by bacteria on the ocean floor  Depletion of oxygen due to stratification

87. A report, ‘’s Hotspots’ finds that eight hundred million South Asians are at risk to see their standards of living and incomes decline as rising temperatures and more erratic rainfalls will cut down crop yields, make water more scarce, and push more people away from their homes to seek safer places. The report is published by ______. (a) World Bank (b) World Economic Forum (c) International Monetary Fund (d) Asian Development Bank

Answer: (a) Explanation: A Bank report, South Asia’s Hotspots, finds that average temperatures in the region have increased in the last sixty years and will continue rising. Rainfall is becoming more erratic: some areas will experience more droughts, others more rain. These changes impact agriculture, health, and productivity. Changes in average weather will create hotspots across South Asia, where the living standards of communities will be negatively impacted

88. Large tracts of natural bamboo forest occur in tropical Asian countries like India. Which among the following regions are rich in bamboo?

Prelim IAS Test Series (2019) – GS Test 11 (17.02.2019) 61 Geography 1 and Current Affairs Nov 2018

(1) Arunachal Pradesh (2) Jammu & Kashmir (3) Assam (4) Andaman & Select the correct answer using the code given below: (a) 1 and 2 only (b) 1 and 3 only (c) 2 and 3 only (d) 1, 3 and 4 only

Answer: (d) Explanation: India is the second richest country in bamboo genetic resources after China. These two countries together have more than half the total bamboo resources globally. The physical geography along with precipitation, temperature and altitudinal variations play significant role towards the diversity and richness of forests of Indian North-Eastern and Eastern part of the country's bamboo flora. Forests and rural areas of North-Eastern and Eastern India comprising the political territories of the States of West Bengal, Sikkim, Arunachal Pradesh, Assam, Manipur, Meghalaya, Nagaland, Mizoram, Tripura have over 50% of the bamboo species of the Indian floristic region. Other areas rich in bamboo are the Andaman & Nicobar Islands, Chhattisgarh, Madhya Pradesh and the Western Ghats.

89. India had launched Project Tiger in 1973 to conserve tigers. Till now, the coverage of “Project Tiger” has increased from 9 reserves to 50 tiger reserves spread across 18 states. Which among the following states do not have a tiger reserve? (1) Punjab (2) Himachal Pradesh (3) Assam (4) Tripura Select the correct answer using the code given below: (a) 1 and 3 only (b) 2 and 3 only (c) 1, 2 and 3 only (d) 1, 2 and 4 only

Answer: (d) Explanation: There are 50 tiger reserves in India which are governed by Project Tiger. Project is administrated by the National Tiger Conservation Authority (NTCA). India is home to 70 percent of tigers in the world. In 2006, there were 1,411 tigers which increased to 1,706 in 2010 and 2,226 in 2014. The total number of wild tigers has risen to 3,890 in 2016 according to World Wildlife Fund and Global Tiger Forum. States like Punjab, Haryana, Himachal Pradesh, Tripura, etc do not have any Tiger reserve as of now. Manas, Orang, Nameri and Kaziranga Tiger Reserves are present in Assam.

Prelim IAS Test Series (2019) – GS Test 11 (17.02.2019) 62 Geography 1 and Current Affairs Nov 2018

90. Red sander wood is a rare kind of sandalwood that is in high demand in international market. In India, Red Sander Wood is protected under the provisions of which of the following? (a) Wildlife Protection Act, 1972 (b) Environment Protection Act, 1986 (c) Indian Forest Act, 1927 (d) Biological Diversity Act, 2002

Answer: (a) Explanation: Red sander wood is a rare kind of sandalwood that is in high demand in international market and costs around Rs. 1,500 to Rs. 2,000 a kg. The major markets for the wood are China, Japan, the , Sri Lanka, Bhutan and Nepal. The wood is protected under the Wildlife Protection Act, 1972. Its export is banned in India. Between 2012-13 to 2014- 15, at least 4658 tonnes of illegal red sander wood has been seized in mainly across Tamil Nadu and Andhra Pradesh where this kind of wood is primarily found.

91. Consider the following statements: (1) National Mission for Manuscripts (NMM) was launched in 2018 by the Ministry of Electronics & IT for documentation, conservation, preservation and digitization of manuscripts. (2) The manuscripts, documented and digitized by the NMM will be made available to researcher and scholars through a Trusted Digital Repository. Which of the statements given above is/are correct? (a) 1 only (b) 2 only (c) Both 1 and 2 (d) Neither 1 nor 2

Answer: (b) Explanation: Statement 1 is incorrect. The National Mission for Manuscripts (NMM) was launched in 2003 by the Ministry of Culture for documentation, conservation, preservation and digitization of manuscripts. Till date NMM has documented 43.16 lakh manuscripts, conserved 434.56 lakh folios of manuscripts, digitized 283 lakh pages of 2.96 lakh manuscripts and published 44 rare and unpublished manuscripts. Statement 2 is correct. The manuscripts documented and digitized by the NMM will be made available to researcher and scholars through a Trusted Digital Repository. The ultimate objective of the Mission is to establish a Digital Manuscripts Repository in which researchers and scholars can view and consult the manuscripts to understand our past in its totality.

92. Which among the following can be considered as suitable items for food fortification? (1) Milk (2) Edible Oil (3) Salt

Prelim IAS Test Series (2019) – GS Test 11 (17.02.2019) 63 Geography 1 and Current Affairs Nov 2018

Select the correct answer using the code given below: (a) 1 only (b) 2 only (c) 1 and 3 only (d) 1, 2 and 3

Answer: (d) Explanation: In order to promote fortification as a means to address micro nutrient deficiencies, the Food Safety and Standards Authority of India (FSSAI) has released benchmarks to fortify the nutritional quality of food items. It has highlighted standards for fortification of food articles namely wheat flour, rice, milk, edible oil and salt with vitamins and minerals. FSSAI may from time to time mandate fortification of any food article specified under the regulations on the directions of the Government of India or on the recommendations of the States/UTs and in consultation with stakeholders. Under Food Safety and Standards (Prohibition and Restriction on Sales) Regulations, 2011, sale of only iodized salt is permitted for direct human consumption. Further, Food Safety and Standards (Food Product Standards and Food Additives) Regulations, 2011 provide that Vanaspati shall contain synthetic Vitamin A. Besides, Ministry of Women and Child Development and Ministry of Human Resource Development have advised the use of double fortified salt (iron and iodine), wheat flour (with iron, folic acid and vitamin B-12) and edible oil with (vitamin A and D) under their Schemes i.e. Integrated Child Development Scheme and Mid-day Meal Scheme.

93. Assumption Island has a great strategic significance. It was in news because at this location (a) India wants to build its naval base. (b) China wants to build its naval base. (c) India wants to build its air base. (d) China wants to build its air base.

Answer: (a) Explanation: Assumption Island is a small island in the Outer Islands of Seychelles north of Madagascar. India has already ratified an agreement to develop a naval base on Assumption Island. India has been working overtime to bolster its naval presence in regional waters to

Prelim IAS Test Series (2019) – GS Test 11 (17.02.2019) 64 Geography 1 and Current Affairs Nov 2018 counter China, which has inaugurated its first overseas military base in Djibouti, near one of the world’s busiest shipping lanes.

94. ‘Earth Bio-genome Project’ plans to record the full DNA sequence of the all eukaryotic species. Which one of the following doesn’t come within the scope of this project? (a) Plants (b) Bacteria (c) Fungi (d) Animals

Answer: (b) Explanation: The Earth Bio Genome Project (EBP), a moonshot for biology, aims to sequence, catalog and characterize the genomes of all of Earth’s eukaryotic biodiversity over a period of ten years. The project encompasses all of the life except simple microbes (bacteria and archaea).

95. Recently, Mangdecchu Project was in news. It is a major ______? (a) hydro-power project in Nepal by India. (b) rail-connectivity project in Nepal by India. (c) hydro-power project in Bhutan by India. (d) rail-connectivity project in Bhutan by India.

Answer: (c) Explanation: India is expected to commission a major hydropower project in Bhutan. The 750- megawatt Mangdechhu hydropower project is being built on River Mengde Chhu which is one of the tributaries of River Manas.

96. National Biodiversity Award, 2018 has been conferred on ‘Singchung Bugun Village Community Reserve Management Committee’ for its efforts in conservation of the Bugun Liocichla bird. ‘Bugun Liocichla’ bird is found in which one of the following states? (a) Assam (b) Arunachal Pradesh (c) Kerala (d) Maharashtra

Answer: (b) Explanation: In 2012, the Government of India, in partnership with UNDP India, initiated the ‘National Biodiversity Awards’ to recognize and honour outstanding models of biodiversity conservation, sustainable use and governance at the grassroots level. The fourth edition of the National Biodiversity Awards on 22 May (2018) on the occasion of the International Day for Biological Diversity recognized the work of local communities committed towards biodiversity conservation in the areas of

Prelim IAS Test Series (2019) – GS Test 11 (17.02.2019) 65 Geography 1 and Current Affairs Nov 2018

 Conservation of Wild and Domesticated Species  Sustainable Use of Biological Resources  Replicable Mechanisms for Access and Benefit Sharing  Best Biodiversity Management Committees National Biodiversity Award, 2018 [Category - Conservation of Wild Species (Institution)] has been conferred on Singchung Bugun Village Community Reserve Management Committee for its efforts in conservation of the Bugun Liocichla bird. Bugun Liocichla is a critically endangered bird. Just 14 to 20 of these birds are known to exist. They are found only in the Singchung village of Arunachal Pradesh, in the northeast of India. Over the years, activities like timber extraction, forest clearance and infrastructure development have threatened their habitat. The Singchung Bugun Village Community Reserve Management Committee, formed in 2012, is using its traditional knowledge to protect the bird and its habitat.

97. Recently, the Maharashtra Legislative Assembly unanimously passed a Bill proposing 16% reservation for Marathas in government jobs and education. With this, 85% of the State’s population will be entitled to constitutional benefits under ______of the Indian Constitution. (a) Article 14(4), 15(4) (b) Article 15(4), 16(4) (c) Article 15(6), 16(6) (d) Article 16(4), 19(4)

Answer: (b) Explanation: Articles 15(4) and 16(4) come under Part III of the Indian Constitution comprising the Fundamental Rights. Articles 15 and 16, which naturally include Articles 15(4) and 16(4), read as follows: 15. Prohibition of discrimination on grounds of religion, race, caste, sex or place of birth- (1) The State shall not discriminate against any citizen on grounds only of religion, race, caste, sex, place of birth or any of them. (2) No citizen shall, on grounds only of religion, race, caste, sex, place of birth or any of them, be subject to any disability, liability, restriction. (3) Nothing in this article shall prevent the State from making any special provision for women and children. (4) Nothing in this article or in clause (2) of Article 29 shall prevent the State from making any special provision for the advancement of any socially or educationally backward classes of citizens or for the Scheduled Castes and the Scheduled Tribes. 16. Equality of opportunity in matters of public employment- (1) There shall be equality of opportunity for all citizens in matters relating to employment or appointment to any office under the State. (4) Nothing in this article shall prevent the State from making any provision for the reservation of appointments or posts in favour of any backward class of citizens which, in the opinion of the State, is not adequately represented in the services under the State.

98. National Anti-profiteering Authority (NAA) has been set up by Government of India recently. It has been set up to address the concerns related to ______

Prelim IAS Test Series (2019) – GS Test 11 (17.02.2019) 66 Geography 1 and Current Affairs Nov 2018

(a) Insider Trading (b) Fugitive Economic Offenders (c) Goods and Services Tax (d) Digital Payment

Answer: (c) Explanation: National Anti-profiteering Authority (NAA), setup under Good and Services Tax (GST) law is mandated to ensure that the benefits of the reduction in GST rates on goods or services are passed on to the ultimate consumers by way of a reduction in prices. This is an institutional mechanism to check the unfair profit-making activities by the trading community. The "anti-profiteering" measures enshrined in the GST law provide an institutional mechanism to ensure that the full benefits of input tax credits and reduced GST rates on supply of goods or services flow to the consumers.

99. 'Akademik Lomonosov' name was in news recently. It is ______? (a) World's first three parent baby. (b) World's first self driving/driverless car (c) World's first floating nuclear power plant. (d) World's first 3d printed sailboat hull

Answer: (c) Explanation: Akademik Lomonosov is the world's first floating nuclear power plant. It is the northernmost nuclear installation in the world. The nuclear plant will reduce the carbon footprint in the Arctic by tens of thousands of tonnes of CO2 emissions every year. Akademic Lomonosov is an unparalleled piece of engineering by Russian scientists. They claim that nuclear power is the only feasible way to reduce the Arctic region’s dependence on coal which leads to excessive carbon dioxide emissions in millions of tonnes and toxic pollution, destroying the region’s fragile ecosystem.

100. Which among the following is the only Indian state where both - Melanistic Tigers and Melanistic Leopards can be spotted? (a) Gujarat (b) West Bengal (c) Maharashtra (d) Odisha

Answer: (d) Explanation: Recently, a rare black panther is spotted in Odisha after 26 years. Black panthers are also called Melanistic Leopards. Like leopards, these animals also have stripes, but they are hardly visible on their black fur due to the excess of melanin. Odisha is the only state in India that is home to melanistic tigers, white tigers and black panthers i.e. Melanistic Leopards.

Prelim IAS Test Series (2019) – GS Test 11 (17.02.2019) 67 Geography 1 and Current Affairs Nov 2018